Vous êtes sur la page 1sur 92

Part 3 : 11/11/10 07:42:55

Question 1 - CMA 1290 4-4 - Decision Making

MultiFrame Company has the following revenue and cost budgets for the two products it sells.

Plastic Frames Glass Frames


Budgeted unit sales 100,000 300,000
Sales price $10.00 $15.00
Direct materials (2.00) (3.00)
Direct labor (3.00) (5.00)
Fixed overhead (3.00) (4.00)
Net income per unit $2.00 $3.00

The budgeted unit sales equal the current unit demand, and total fixed overhead for the year is budgeted at $975,000.
Assume that the company plans to maintain the same proportional mix. In numerical calculations, MultiFrame rounds
to the nearest cent and unit.

The total number of units MultiFrame needs to produce and sell to break even is

A. 150,000 units.
B. 354,545 units.
C. 300,000 units.
D. 177,273 units.

A.

The breakeven calculation for a multi-product company is a bit more complicated than the breakeven
calculation for a single product company. The first step is to calculate the composite unit contribution
margin.

For 100,000 units of plastic frames and 300,000 units of glass frames, the sales ratio is 1:3. The unit
contribution margin for plastic frames is $10 − $2 DM − $3 DL, or $5. The unit contribution margin for glass
frames is $15 − $3 DM − $5 DL, or $7. Therefore, the composite UCM is ($5 × 1) + ($7 × 3), or $26. Fixed costs
are $975,000, so the breakeven point at a ratio of 1:3 is $975,000 ÷ $26, or 37,500 composite units. These will
consist of (37,500 × 1) plastic frames and (37,500 × 3) glass frames, or 112,500 glass frames, or 150,000 units
in total.

B. See correct answer.

C. See correct answer.

D. See correct answer.

Question 2 - CMA 687 5-25 - Decision Making

Gleason Co. has two products, a frozen dessert and ready-to-bake breakfast rolls, ready for introduction. However,
plant capacity is limited, and only one product can be introduced at present. Therefore, Gleason has conducted a
market study, at a cost of $26,000, to determine which product will be more profitable. The results of the study follow.

Sales of Desserts Sales of Rolls


at $1.80/unit at $1.20/unit
Volume Probability Volume Probability
250,000 .30 200,000 .20

(c) HOCK international, page 1


Part 3 : 11/11/10 07:42:55

300,000 .40 250,000 .50


350,000 .20 300,000 .20
400,000 .10 350,000 .10

The costs associated with the two products have been estimated by Gleason's cost accounting department and are
shown as follows.

Dessert Rolls
Ingredients per unit $ .40 $ .25
Direct labor per unit .35 .30
Variable overhead per unit .40 .20
Production tooling* 48,000 25,000
Advertising 30,000 20,000

*Gleason treats production tooling as a current operating expense rather than capitalizing it as a fixed asset.

The cost incurred by Gleason for the market study is a(n)

A. Sunk cost.
B. Incremental cost.
C. Opportunity cost.
D. Prime cost.

A. Since the market study cost has been already incurred it will not have any relevance to future management
decisions.

B. Incremental (differential) cost are additional cost that result when production, or some other factor is increased. The
market study has already been incurred, so it is a sunk cost.

C. Opportunity cost is the cost of not selecting the best alternative. This does not relate to the cost of the market study.

D. Prime cost include direct labor and direct material. Marketing study is a sunk cost.

Question 3 - CIA 1193 IV-12 - Decision Making

Total production costs of prior periods for a company are listed as follows. Assume that the same cost behavior
patterns can be extended linearly over the range of 3,000 to 35,000 units and that the cost driver for each cost is the
number of units produced.

Production (units/month): 3,000 9,000 16,000 35,000


Cost X: $23,700 $52,680 $86,490 $178,260
Cost Y: 47,280 141,840 252,160 551,600

The company is concerned about its current operating performance that is summarized as follows.

Sales ($12.50 per unit) $300,000


Variable costs 180,000
Net operating loss (40,000)

How many additional units should have been sold in order for the company to break even?

A. 8,000
B. 32,000
C. 12,800

(c) HOCK international, page 2


Part 3 : 11/11/10 07:42:55

D. 16,000

A.

First, we need to find the breakeven point in units, and then compare that with the actual units sold to
determine how many additional units should have been sold in order to break even.

The formula for the breakeven point in units is:

FC / Unit Contribution Margin = BEP in units

So we need to find fixed costs and the unit contribution margin.

Given a loss of $40,000, then fixed costs must be $160,000. The formula is:

$300,000 sales - $180,000 variable costs - X = ($40,000)

$120,000 - X = ($40,000)

X = $160,000

Next, find the unit contribution margin. The total contribution margin is $300,000 - $180,000, or $120,000. The
sale price (given in the problem) was $12.50 per unit. With sales of $300,000 at $12.50 per unit, a total of
24,000 units was sold ($300,000 / $12.50). Therefore, the Unit Contribution Margin is $120,000 / 24,000, or $5.

Now, we can calculate the BEP in units.

BEP in units = $160,000 / $5, or 32,000 units

The final step is to determine the additional number of units that would have been required, over and above
what was actually sold, in order to break even. We have already calculated that 24,000 units were actually
sold and that 32,000 units would have been needed to break even. Thus, 8,000 additional units should have
been sold in order for the company to break even (32,000 - 24,000).

B. 32,000 is the total units needed in order for the company to break even. However, the question asks for the
additional units needed in order for the company to break even. So the correct answer is the difference between the
number of units that actually were sold and the breakeven volume.

C. See correct answer.

D. See correct answer.

Question 4 - CIA 1188 IV-16 - Decision Making

Positive net income is shown on a cost-volume-profit chart when the

A. Total expense line exceeds the total sales revenue line.


B. Total variable expense line exceeds the total fixed expense line.
C. Total sales revenue line exceeds the total expense line.
D. Total sales revenue line exceeds the total fixed expense line.

A. At this point the company would have a negative profit.

B. This answer is only looking at expenses and not comparing with sales revenue.

(c) HOCK international, page 3


Part 3 : 11/11/10 07:42:55

C. The CVP chart identifies the variable and fixed cost. Total variable and fixed costs will equal total costs.
Any time total sales revenue exceed total expenses a company will have positive net income.

D. This represents the contribution margin. Must compare sales revenue with total expenses (variable and fixed).

Question 5 - CMA 1290 4-11 - Decision Making

The relevance of a particular cost to a decision is determined by the

A. Potential effect on the decision.


B. Riskiness of the decision.
C. Accuracy and verifiability of the cost.
D. Size of the cost.

A. The relevance of a cost depends on whether it varies among the related options.

B. The riskiness of a project is not relevant if it does not affect the decision process.

C. Some cost might be considered relevant even if they might only be estimates.

D. The size of the cost is not relevant if the cost does not affect the decision process.

Question 6 - CMA 694 4-21 - Decision Making

Management accountants are frequently asked to analyze various decision situations, including the following:

I. The cost of a special device that is necessary if a special order is accepted.

II. The cost proposed annually for the plant service for the grounds at corporate headquarters.

III. Joint production costs incurred, to be considered in a sell-at-split versus a process-further decision.

IV. The costs associated with alternative uses of plant space, to be considered in a make/buy decision.

V. The cost of obsolete inventory acquired several years ago, to be considered in a keep-versus-disposal decision.

The cost described in situation II is a

A. Relevant cost.
B. Discretionary cost.
C. Imputed cost.
D. Prime cost.

A. Relevant costs are those costs that will differ among the decision options. Situation II would not be considered a
relevant costs.

B. Discretionary cost have no relationship between input-output. This is best described in situation II since
the annual cost of plant service could be subjected to periodic changes.

C. Imputed costs are those costs that are not paid in cash, but are implied. The best example of an imputed cost is the
interest charge on equity capital.

(c) HOCK international, page 4


Part 3 : 11/11/10 07:42:55

D. Prime costs include direct labor, and direct materials. This is not described in situation II.

Question 7 - CMA 694 4-25 - Decision Making

Condensed monthly operating income data for Korbin Inc. for May follows:

Urban Store Suburban Store Total


Sales $80,000 $120,000 $200,000
Variable costs 32,000 84,000 116,000
Contribution margin 48,000 36,000 84,000
Direct fixed costs 20,000 40,000 60,000
Store segment margin 28,000 (4,000) 24,000
Common fixed cost 4,000 6,000 10,000
Operating income 24,000 (10,000) 14,000

Additional information regarding Korbin's operations follows:


One-fourth of each store's direct fixed costs would continue if either store were closed.
Korbin allocates common fixed costs to each store on the basis of sales dollars.
Management estimates that closing the Suburban Store would result in a 10% decrease in the Urban Store's
sales, while closing the Urban Store would not affect the Suburban Store's sales.
The operating results for May are representative of all months.

A decision by Korbin to close the Suburban Store would result in a monthly increase (decrease) in Korbin's operating
income of

A. $(10,800)
B. $(1,200)
C. $(6,000)
D. $4,000

A. Total operating income before closing the Suburban store is $14,000. Korbin has concluded that the
following information will affect its net operating income: 1. Sales (decrease by 10%) = $72,000 2. VC (also
decrease by 10%) = $28,800 3. Contribution Margin = $43,200 4. Direct fixed cost (1/4 of Suburban fixed cost
would continue) = $30,000 5. Store segment margin = $13,200 6. Common cost = $10,000 7. Operating Income
= $3,200 If Korbin closes the store then operating income will decrease to $3,200. The net affect to Korbin will
be to decrease net operating income by $10,800 ($3,200 - $14,000).

B. The correct answer is a decrease in operating income of $10,800.

C. The correct answer is a decrease in operating income of $10,800.

D. The correct answer is a decrease in operating income of $10,800.

Question 8 - CMA 692 4-29 - Decision Making

Barnes Corporation manufactures skateboards and is in the process of preparing next year's budget. The pro forma
income statement for the current year is presented as follows.

Sales $1,500,000

(c) HOCK international, page 5


Part 3 : 11/11/10 07:42:55

Cost of sales:
Direct materials 250,000
Direct labor 150,000
Variable overhead 75,000
Fixed overhead 100,000
Gross profit 925,000
Selling and G&A Variable 200,000
Selling and G&A Fixed 250,000
Operating income 475,000

The breakeven point (rounded to the nearest dollar) for Barnes Corporation for the current year is

A. $146,341.
B. $181,818.
C. $729,730.
D. $636,364.

A. See correct answer.

B. See correct answer.

C. See correct answer.

D. The answer for this problem is found by dividing Fixed Cost by contribution margin percentage. Sales
-$1,500,000 Less: VC -$675,000 CM -$825,000 CM %55% $350,000 fixed costs =55% =$636,364 breakeven point

Question 9 - CMA 1288 5-13 - Decision Making

Management accountants are frequently asked to analyze various decision situations including the following.

1. Alternative uses of plant space, to be considered in a make/buy decision.

2. Joint production costs incurred, to be considered in a sell-at-split versus a process-further decision.

3. Research and development costs incurred in prior months, to be considered in a product-introduction decision.

4. The cost of a special device that is necessary if a special order is accepted.

5. The cost of obsolete inventory acquired several years ago, to be considered in a keep-versus-disposal decision.

The costs described in situations 2, 3, and 5 are

A. Sunk costs.
B. Differential costs.
C. Relevant costs.
D. Discretionary costs.

A. The costs described in 2, 3 and 5, are sunk costs. Sunk costs are those costs that have already been
incurred and are not relevant to management's future decisions. Joint costs are not relevant as are R&D, or
obsolete inventory.

B. Differential (incremental) costs are additional costs that result when production, or some other factor is increased.

C. Relevant costs are future costs that are expected to vary will activity.

(c) HOCK international, page 6


Part 3 : 11/11/10 07:42:55

D. Discretionary costs are those costs that have no direct input/output relationship.

Question 10 - CMA 1296 4-1 - Decision Making

The term that best refers to past costs that have been incurred and are not relevant to any future decisions is

A. Incurred marginal costs.


B. Discretionary costs.
C. Sunk costs.
D. Full absorption costs.

A. Marginal costs change with increases/decreases in production levels. These costs are relevant to future decisions.

B. Discretionary cost have no correlation with input-output and they may or may not be relevant to future decisions.

C. Sunk cost are past cost that will not change with related options. Sunk cost are irrelevant to management's
future decisions.

D. Absorption costing includes direct material, direct labor and factory overhead. These costs may or may not be
relevant to any future decisions.

Question 11 - CMA 694 4-20 - Decision Making

Management accountants are frequently asked to analyze various decision situations, including the following:

I. The cost of a special device that is necessary if a special order is accepted.

II. The cost proposed annually for the plant service for the grounds at corporate headquarters.

III. Joint production costs incurred, to be considered in a sell-at-split versus a process-further decision.

IV. The costs associated with alternative uses of plant space, to be considered in a make/buy decision.

V. The cost of obsolete inventory acquired several years ago, to be considered in a keep-versus-disposal decision.

The costs described in situations III and V are

A. Sunk costs.
B. Prime costs.
C. Discretionary costs.
D. Imputed costs.

A. Situation III and V are describing sunk costs. Sunk costs are costs that are not relevant to decisions;
decision whether to process beyond its current state, or what to do with obsolete inventory.

B. Prime cost include only Direct Material and Direct Labor.

C. Discretionary costs are incurred costs that have no relationship between input-output (advertising).

D. An imputed cost is a cost that does not really exist, but is needed for the decision process. The common example
of imputed cost is the interest cost of equity capital.

(c) HOCK international, page 7


Part 3 : 11/11/10 07:42:55

Question 12 - HOCK DA 2 - Decision Making

Stark Rehabilitation Hospital is an inpatient institution for severely injured patients who need intense therapy to
recover normal functioning. The hospital has a maximum capacity of 130 patients. The hospital employs nurses,
doctors and physical therapists. Expenses are as follows:

Medical staff:
- 1 nurse for every 20 patients, average cost $66,000 per year per nurse
- 1 doctor for every 60 patients, average cost $150,000 per year per doctor
- 1 physical therapist for every 10 patients, average cost $80,000 per year per therapist

The cost per patient per day for meals, medicine and supplies averages $60 per day per patient.

Utilities cost a base amount of $2,500 per month plus an average of $200 per patient per month.

Medical and exercise equipment, occupancy expense and administrative expense total $7,700,000 per year.

The hospital patient census (i.e., number of patients at any one time) varies from a minimum of 100 patients to fully
occupied at 130 patients.

Stark charges $300 per day per patient, and this includes the room, meals, medicine, supplies and all services.

What are Stark's variable costs?

A. Medical staff
B. Medical and exercise equipment, occupancy expense and administative expense
C. Utilities
D. Meals, medicine and supplies

A. Variable costs change in total in proportion to changes in the level of activity. While it could be argued that the
expenses of the medical staff behave in this way to a degree, there is a better answer choice that more closely fits this
description.

B. Variable costs change in total in proportion to changes in the level of activity. Medical and exercise equipment,
occupancy expense and administrative expenses do not change in total in proportion to changes in the level of activity.

C. Variable costs change in total in proportion to changes in the level of activity. While it could be argued that utilities
cost as it is described in this question behaves in this way, there is a better answer choice that more closely fits this
description.

D. Variable costs change in total in proportion to changes in the level of activity. Meals, medicine and
supplies fit this description.

Question 13 - CMA 693 4-3 - Decision Making

Delphi Company has developed a new project that will be marketed for the first time during the next fiscal year.
Although the Marketing Department estimates that 35,000 units could be sold at $36 per unit, Delphi's management
has allocated only enough manufacturing capacity to produce a maximum of 25,000 units of the new product annually.
The fixed costs associated with the new product are budgeted at $450,000 for the year, which includes $60,000 for
depreciation on new manufacturing equipment. Data associated with each unit of product are presented as follows.
Delphi is subject to a 40% income tax rate.

(c) HOCK international, page 8


Part 3 : 11/11/10 07:42:55

Variable Costs
Direct material $7.00
Direct labor 3.50
Manufacturing overhead 4.00
Total variable manufacturing cost 14.50
Selling expenses 1.50
Total variable cost $16.00

Delphi Company's management has stipulated that it will not approve the continued manufacture of the new product
after the next fiscal year unless the after-tax profit is at least $75,000 the first year. The unit selling price to achieve
this target profit must be at least

A. $34.60.
B. $37.00.
C. $39.00.
D. $36.60.

A. See correct answer.

B. See correct answer.

C. Need to set the problem up algebraically. 25,000 units (X - $16 variable cost) - $450,000 fixed cost =
$125,000 before-tax profit 25,000X - $400,000 - $450,000 = $125,000 25,000X = $400,000 +$450,000 + $125,000
25,000X = $975,000 = $39

D. See correct answer.

Question 14 - CIA 594 III-42 - Decision Making

The following data pertains to XYZ Company for the current year of operations.

Total Cost Unit Cost


Sales (40,000 units) $1,000,000 $25
Raw materials 160,000 4
Direct labor 280,000 7
Factory overhead:
Variable 80,000 2
Fixed 360,000
Selling and general expenses:
Variable 120,000 3
Fixed 225,000

How many units does XYZ Company need to produce and sell to make a before-tax profit of 10% of sales?

A. 25,000 units.
B. 65,000 units.
C. 36,562 units.
D. 90,000 units.

A. See correct answer.

B. 65,000 units is the breakeven volume.

C. See correct answer.

(c) HOCK international, page 9


Part 3 : 11/11/10 07:42:55

D.

The formula to calculate the answer to this problem would be:

5X − 16X − 585,000 = (25X * .10)


9X − 585,000 = 2.5X
6.5X − 585,000 = 0
6.5X = 585,000
X = 90,000 units

Question 15 - CIA 596 III-84 - Decision Making

A company that sells its single product for $40 per unit uses cost-volume-profit analysis in its planning. The company's
after-tax net income for the past year was $1,188,000 after applying an effective tax rate of 40%. The projected costs
for manufacturing and selling its single product in the coming year are shown below.

Variable cost per unit:


Direct material $5.00
Direct labor 4.00
Manufacturing overhead 6.00
Selling and administrative costs 3.00
Total variable cost per unit $18.00
Annual fixed operating costs:
Manufacturing overhead $6,200,000
Selling and administrative costs 3,700,000
Total annual fixed cost $9,900,000

The dollar sales volume required in the coming year to earn the same after-tax net income as the past year is

A. $23,400,000
B. $26,400,000
C. $21,600,000
D. $20,160,000

A. See correct answer.

B. See correct answer.

C. The company desires to maintain the same profit level as the year before. If the after-tax profit was
$1,188,000, then the pre-tax profit would be $1,980,000 ($1,188,000 / (1 - 40%). The answer to find the dollar
sales volume is to divide total Fixed Cost, plus pre-tax profit by unit contribution margin percentage. Sales
-$40 Less: VC -$18 CM -$22 CM% -55% $9,900,000 + $1,980,000 (pre-tax profit)=55% =$21,600,000

D. See correct answer.

Question 16 - CMA 691 4-10 - Decision Making

Sunk costs

A. Are substitutes for opportunity costs.

(c) HOCK international, page 10


Part 3 : 11/11/10 07:42:55

B. In and of themselves are not relevant to decision making.


C. Are fixed costs.
D. Are relevant to decision making.

A. Opportunity cost and sunk cost have nothing to do with each other.

B. Sunk cost are past incurred cost that are irrelevant in the decision making process.

C. Sunk cost might include both fixed and variable cost.

D. Sunk cost are irrelevant to the decision making process.

Question 17 - CMA 1292 4-2 - Decision Making

What is the opportunity cost of making a component part in a factory given no alternative use of the capacity?

A. The variable manufacturing cost of the component.


B. The total variable cost of the component.
C. The total manufacturing cost of the component.
D. Zero.

A. Opportunity cost is a foregone benefit (inflow), not a cash outflow

B. Opportunity cost is a foregone benefit (inflow), not a cash outflow

C. Opportunity cost is a foregone benefit (inflow), not a cash outflow

D. Opportunity cost is the benefit to income (inflow) that is lost by not using a limited resource for its best
alternative use. In this question there is no alternative use of the capacity, so opportunity cost would be zero.

Question 18 - CMA 1285 5-29 - Decision Making

The opportunity cost of making a component part in a factory with no excess capacity is the

A. Total manufacturing cost of the component.


B. Net benefit forgone from the best alternative use of the capacity required.
C. Cost of the production given up in order to manufacture the component.
D. Variable manufacturing cost of the component.

A. Opportunity cost is a foregone benefit (inflow), not a cash outflow

B. Opportunity cost is the benefit to income (inflow) that is lost by not using a limited resource for its best
alternative use. Even though there is no excess capacity there is still an alternative use of the capacity.

C. Opportunity cost is a foregone benefit (inflow), not a cash outflow

D. Opportunity cost is a foregone benefit (inflow), not a cash outflow

Question 19 - CIA 586 IV-8 - Decision Making

(c) HOCK international, page 11


Part 3 : 11/11/10 07:42:55

A company sells two products, X and Y. The sales mix consists of a composite unit of 2 units of X for every 5 units of
Y (2:5). Fixed costs are $49,500. The unit contribution margins for X and Y are $2.50 and $1.20, respectively.

Considering the company as a whole, the number of composite units to break even is

A. 8,250
B. 1,650
C. 22,500
D. 4,500

A. See correct answer.

B. See correct answer.

C. See correct answer.

D. The breakeven analysis is more difficult for a multi-product company. The formula is found by dividing
fixed cost by the composite contribution margin. The composite contribution margin in this problem is $11
[2($2.50) + 5($1.20)]. The breakeven point = $49,500 / $11 = 4,500 units

Question 20 - CMA 1286 5-17 - Decision Making

Hermo Company has just completed a hydro-electric plant at a cost of $21,000,000. The plant will provide the
company's power needs for the next 20 years. Hermo will use only 60% of the power output annually. At this level of
capacity, Hermo's annual operating costs will amount to $1,800,000, of which 80% are fixed.

Quigley Company currently purchases its power from MP Electric at an annual cost of $1,200,000. Hermo could
supply this power, thus increasing output of the plant to 90% of capacity. This would reduce the estimated life of the
plant to 14 years.

If Hermo decides to supply power to Quigley, it wants to be compensated for the decrease in the life of the plant and
the appropriate variable costs. Hermo has decided that the charge for the decreased life should be based on the
original cost of the plant calculated on a straight-line basis. The minimum annual amount that Hermo would charge
Quigley would be

A. $450,000.
B. $990,000.
C. $630,000.
D. Some amount other than those given.

A. This is incorrect because this is only the increase in annual FC that would be incurred if expanding the capacity,
thereby, shortening the depreciable life ($21,000,000 / 14 year life - $21,000,000 / 20 year life).

B. This answer was found by taking the increase in FC and adding the total VC of operating at 90% capacity
($450,000 + $540,000).

C. This calculation has to include any additional increases in FC and VC that arise from the sale. The
increased FC is $450,000, which was calculated by taking the difference between depreciation over a 14 year
life and 20 year life ($1,500,000 - $1.050,000). If the cost of operating at 60% capacity is $360,000 then
increasing capacity by 50% (to 90%) would increase costs by $180,000. Therefore, total increased cost would
be $630,000 ($450,000 + $180,000).

D. The correct answer is B.

(c) HOCK international, page 12


Part 3 : 11/11/10 07:42:55

Question 21 - CMA 690 5-15 - Decision Making

Madengrad Company manufactures a single electronic product called Precisionmix. This unit is a batch-density
monitoring device attached to large industrial mixing machines used in flour, rubber, petroleum, and chemical
manufacturing. Precisionmix sells for $900 per unit. The following variable costs are incurred to produce each
Precisionmix device:

Direct labor $180


Direct materials 240
Factory overhead 105
Total variable production costs 525
Marketing costs 75
Total variable costs $600

Madengrad's income tax rate is 40%, and annual fixed costs are $6,600,000. Except for an operating loss incurred in
the year of incorporation, the firm has been profitable over the last 5 years.

Assume a 10% increase in annual fixed costs, a 20% unit cost increase for direct labor, and a reduction in unit
material costs of 25%, with no change in selling price. After incorporating these changes, Madengrad Company's
contribution margin would be

A. 36%
B. 34%
C. 64%
D. 69%

A. The 20% unit cost increase in direct labor and a reduction of material cost of 25%, will cause a unit cost
decrease in variable cost of $24. Contribution margin after adjustments is $324 per unit ($300 contribution
margin before changes + $24 reduction in variable cost). The contribution margin percentage is found by
dividing per unit contribution margin by the per unit sales price ($324 / $900).

B. See correct answer.

C. See correct answer.

D. See correct answer.

Question 22 - CMA 1292 4-5 - Decision Making

Richardson Motors uses 10 units of Part No. T305 each month in the production of large diesel engines. The cost to
manufacture one unit of T305 is presented as follows.

Direct materials $ 2,000


Materials handling (20% of direct materials cost) 400
Direct labor 16,000
Manufacturing overhead (150% of direct labor) 24,000
Total manufacturing cost $42,400

Materials handling, which is not included in manufacturing overhead, represents the direct variable costs of the
receiving department that are applied to direct materials and purchased components on the basis of their cost.
Richardson's annual manufacturing overhead budget is one-third variable and two-thirds fixed. Simpson Castings, one
of Richardson's reliable vendors, has offered to supply T305 at a unit price of $30,000.

(c) HOCK international, page 13


Part 3 : 11/11/10 07:42:55

Assume Richardson Motors could use the idle capacity to manufacture another product that would contribute
$104,000 per month. If Richardson chooses to manufacture the ten T305 units in order to maintain quality control,
Richardson's opportunity cost is

A. $88,000.
B. $68,000.
C. $(96,000).
D. $8,000.

A. This answer uses a variable overhead amount of 2/3 of the total overhead to calculate the avoidable variable
manufacturing expense, instead of 1/3.

B. This answer fails to include the materials handling cost in the calculation of the net cost to purchase the T305 from
an outside supplier.

C. This is the additional cost to purchase the units outside. However, this answer ignores the rental opportunity of
$104,000.

D.

Total per unit manufacturing cost for the T305 is $42,400. Since the overhead of $24,000 is 1/3 variable and 2/3
fixed, $16,000 of that would continue. Thus, the avoidable variable manufacturing cost per unit is $26,400.

If Richardson Motors purchases the units from Simpson Castings then the per unit variable cost will be
$36,000 ($30,000 purchase price + $6,000 material handling cost applied [20% X $30,000 per unit]. Therefore,
by purchasing from Simpson Castings the per unit cost of the T305 component will increase $9,600 ($36,000 -
$26,400). By purchasing 10 units per month, total monthly cost would increase by $96,000 ($9,600 X 10).

If Richardson Motors could manufacture another product that would contribute $104,000 per month, then the
opportunity cost would be $8,000 ($104,000 - $96,000).

Question 23 - CIA 593 IV-11 - Decision Making

A company has sales of $500,000, variable costs of $300,000, and pretax profit of $150,000. If the company
increased the sales price per unit by 10%, reduced fixed costs by 20%, and left variable cost per unit unchanged, what
would be the new breakeven point in sales dollars?

A. $100,000
B. $125,000
C. $88,000
D. $110,000

A. See correct answer.

B. See correct answer.

C. If needing to find the breakeven point need to first calculate what fixed costs are. Based on the given
information fixed costs must be $50,000 ($500,000 - $300,000 - 150,000). In the coming year is fixed costs are
reduced by 20%, fixed costs will be $40,000 ($50,000 X 80%). The contribution margin given the stated
changes will be $250,000 [($500,000 X 110%) - $300,000] The contribution margin rate is 45.45% ($250,000 /
$550,000). The formula for breakeven sales revenue is: = Fixed costs / contribution margin rate = $40,000 /
45.45% = $88,000 breakeven sales revenue

D. See correct answer.

(c) HOCK international, page 14


Part 3 : 11/11/10 07:42:55

Question 24 - CMA 690 5-23 - Decision Making

When a decision is made in an organization, it is selected from a group of alternative courses of action. The loss
associated with choosing the alternative that does not maximize the benefit is the

A. Opportunity cost.
B. Incremental cost.
C. Net realizable value.
D. Expected value.

A. By definition opportunity cost is the benefit to income (inflow) that is lost by not using a limited resource
for its best alternative use.

B. Incremental (differential) costs are additional costs that result when production, or some other factor is increased.

C. Net realizable value is value of an asset less any disposal cost.

D. The expected value of a product is the weighted average of potential outcomes. It is calculated by taking the
expected volume of sales and multiplying by the estimated probabilities for each potential outcome and adding
together.

Question 25 - CMA 1291 4-13 - Decision Making

Siberian Ski Company recently expanded its manufacturing capacity, which will allow it to produce up to 15,000 pairs
of cross-country skis of the mountaineering model or the touring model. The Sales Department assures management
that it can sell between 9,000 pairs and 13,000 pairs of either product this year. Because the models are very similar,
Siberian Ski will produce only one of the two models.
The following information was compiled by the Accounting Department.

Per Unit (Pair) Data


Mountaineering Touring
Selling price $88.00 $80.00
Variable costs 52.80 52.80

Fixed costs will total $369,600 if the mountaineering model is produced but will be only $316,800 if the touring model
is produced. Siberian Ski is subject to a 40% income tax rate.

If Siberian Ski Company desires an after-tax net income of $24,000, how many pairs of touring model skis will the
company have to sell?

A. 13,118 pairs.
B. 4,460 pairs.
C. 12,529 pairs.
D. 13,853 pairs.

A. The company desires an after-tax income of $24,000 the formula to calculate the sales breakeven even
volume is: Total fixed cost + Profit / unit contribution margin = $316,800 (fixed costs) + $40,000 before-tax
profit ($24,000 / (1-40% tax rate))/ $27.20 contribution margin per unit ($80 sales price - $52.80 variable cost) =
13,118 units

B. See correct answer.

(c) HOCK international, page 15


Part 3 : 11/11/10 07:42:55

C. See correct answer.

D. See correct answer.

Question 26 - CMA 1287 5-28 - Decision Making

Leland Manufacturing uses 10 units of Part Number KJ37 each month in the production of radar equipment. The unit
cost to manufacture 1 unit of KJ37 is presented below.

Direct materials $ 1,000


Materials handling (20% of direct material cost) 200
Direct labor 8,000
Manufacturing overhead (150% of direct labor) 12,000
Total manufacturing cost $21,200

Material handling represents the direct variable costs of the Receiving Department that are applied to direct materials
and purchased components on the basis of their cost. This is a separate charge in addition to manufacturing
overhead. Leland's annual manufacturing overhead budget is one-third variable and two-thirds fixed. Scott Supply,
one of Leland's reliable vendors, has offered to supply Part Number KJ37 at a unit price of $15,000.

If Leland purchases the KJ37 units from Scott, the capacity Leland used to manufacture these parts would be idle.
Should Leland decide to purchase the parts from Scott, the unit cost of KJ37 would

A. Decrease by $6,200.
B. Decrease by $3,200.
C. Change by some amount other than those given.
D. Increase by $4,800.

A. Assumes that there are no other cost involved in the decision besides the initial purchase price of $15,000 ($21,200
- $15,000 = $6,200). Other relevant cost would include unavoidable variable cost and unavoidable fixed cost.

B. This answer does not include unavoidable fixed cost of $8,000.

C. See correct answer.

D. Total manufacturing cost for Leland is $21,200. If Leland purchases the units from Scott then the per unit
cost will be $26,000 [$15,000 purchase price + $3,000 unavoidable variable cost (20% handling cost X $15,000)
+ $8,000 unavoidable fixed cost ($12,000 fixed cost X 2/3)]. By purchasing from Scott the per unit cost
increase of component KJ37 will be $4,800 ($21,200 - $26,000).

Question 27 - CIA 1193 IV-15 - Decision Making

A printing company is considering replacing an old printing press. The old printing press has a book value of $24,000
and a trade-in value of $14,000. A new printing press would cost $85,000 after trade-in of the old press. It is estimated
that the new printing press would reduce operating costs by $20,000 per year. If the company decides not to purchase
the new press, the $85,000 could instead be used to retire debt that is currently costing $9,000 per year in interest.
Which of the given is an example of a sunk cost?

A. The interest on the existing debt.


B. The book value of the old printing press.

(c) HOCK international, page 16


Part 3 : 11/11/10 07:42:55

C. The estimated reduction in operating costs.


D. The trade-in value of the old printing press.

A. The interest on debt is an avoidable cost that is relevant to the decision process.

B. A sunk cost is a cost that has already been incurred and is not relevant to the decision process. Sunk cost
are irrelevant.

C. The reduction of operating expenses is the difference between keeping the old printing press and purchasing the
new one.

D. The trade-in value of the old printing press will affect the decision process, so therefore, is relevant.

Question 28 - CMA 696 4-16 - Decision Making

In a decision analysis situation, which one of the following costs is generally not relevant to the decision?

A. Incremental cost.
B. Avoidable cost.
C. Differential cost.
D. Historical cost.

A. Incremental (differential) costs are additional costs that result when production, or some other factor is increased.
These cost are relevant to the decision making process.

B. Avoidable cost are relevant to decision making process. In the decision making process need to look at all
avoidable (variable and fixed) cost that will vary among the related options.

C. Incremental (differential) costs are additional costs that result when production, or some other factor is increased.
These cost are relevant to the decision making process.

D. Historical cost are sunk cost that are not relevant to the decision making process. In the decision making
process you want to look only at cost that differ among the related options.

Question 29 - CMA 1296 4-7 - Decision Making

Whitehall Corporation produces chemicals used in the cleaning industry. During the previous month, Whitehall
incurred $300,000 of joint costs in producing 60,000 units of AM-12 and 40,000 units of BM-36. Whitehall uses the
units-of-production method to allocate joint costs. Currently, AM-12 is sold at split-off for $3.50 per unit. Flank
Corporation has approached Whitehall to purchase all of the production of AM-12 after further processing. The further
processing will cost Whitehall $90,000.

Concerning AM-12, which one of the following alternatives is most advantageous?

A. Whitehall should process further and sell to Flank if the total selling price per unit after further processing is greater
than $3.00, which covers the joint costs.
B. Whitehall should process further and sell to Flank if the total selling price per unit after further processing is greater
than $5.25, which maintains the same gross profit percentage.
C. Whitehall should continue to sell at split-off unless Flank offers at least $4.50 per unit after further processing,
which covers Whitehall's total costs.
D. Whitehall should process further and sell to Flank if the total selling price per unit after further processing is greater
than $5.00.

(c) HOCK international, page 17


Part 3 : 11/11/10 07:42:55

A. Joint cost are irrelevant to the decision.

B. Any price greater than $5.00 per unit would increase the profit margin.

C. The minimum price that Whitehall would accept is $5.00 per unit ($3.50 sell price at split-off + $1.50 additional cost).

D. If we know that AM-12 can be sold for $3.50 per unit at split-off then joint cost are irrelevant to the decision
whether to process further or not. If further processing will cost $90,000 then the additional cost of
processing would be $1.50 per unit ($90,000 / 60,000 units of AM-12). Therefore, the minimum price that
Whitehall would accept must be a price no less than $5.00 per unit ($3.50 + $1.50).

Question 30 - HOCK DA 1 - Decision Making

Stark Rehabilitation Hospital is an inpatient institution for severely injured patients who need intense therapy to
recover normal functioning. The hospital has a maximum capacity of 130 patients. The hospital employs nurses,
doctors and physical therapists. Expenses are as follows:

Medical staff:
- 1 nurse for every 20 patients, average cost $66,000 per year per nurse
- 1 doctor for every 60 patients, average cost $150,000 per year per doctor
- 1 physical therapist for every 10 patients, average cost $80,000 per year per therapist

The cost per patient per day for meals, medicine and supplies averages $60 per day per patient.

Utilities cost a base amount of $2,500 per month plus an average of $200 per patient per month.

Medical and exercise equipment, occupancy expense and administrative expense total $7,700,000 per year.

The hospital patient census (i.e., number of patients at any one time) varies from a minimum of 100 patients to fully
occupied at 130 patients.

Stark charges $300 per day per patient, and this includes the room, meals, medicine, supplies and all services.

What are Stark's fixed costs?

A. Medical and exercise equipment, occupancy expense and administrative expense


B. Medical staff
C. Meals, medicine and supplies
D. Utilities

A. Fixed costs are costs which do not change in total as long as the volume remains within the relevant
range. Medical and exercise equipment, occupancy expense and administrative expenses do not change in
total as long as the volume remains within the relevant range.

B. Fixed costs are costs which do not change in total as long as the volume remains within the relevant range.
Although it could be argued that the small ranges for each level of medical staffing are relevant ranges, there is a
better answer choice that more closely fits this description.

C. Fixed costs are costs which do not vary with activity. Meals, medicine and supplies vary with activity.

D. Fixed costs are costs which do not vary with activity. Utilities vary with activity.

(c) HOCK international, page 18


Part 3 : 11/11/10 07:42:55

Question 31 - CMA 679 5-27 - Decision Making

Moorehead Manufacturing Company produces two products for which the following data have been tabulated. Fixed
manufacturing cost is applied at a rate of $1.00 per machine hour.

Per Unit XY-7 BD-4


Selling price $4.00 $3.00
Variable manufacturing cost $2.00 $1.50
Fixed manufacturing cost $ .75 $ .20
Variable selling cost $1.00 $1.00

The sales manager has had a $160,000 increase in the budget allotment for advertising and wants to apply the money
to the most profitable product. The products are not substitutes for one another in the eyes of the company's
customers.

Suppose Moorehead has only 100,000 machine hours that can be made available to produce additional units of XY-7
and BD-4. If the potential increase in sales units for either product resulting from advertising is far in excess of this
production capacity, which product should be advertised and what is the estimated increase in contribution margin
earned?

A. Product BD-4 should be produced, yielding a contribution margin of $250,000.


B. Product BD-4 should be produced, yielding a contribution margin of $187,500.
C. Product XY-7 should be produced, yielding a contribution margin of $133,333.
D. Product XY-7 should be produced, yielding a contribution margin of $75,000.

A. In this problem it is necessary to optimize the contribution margin of a scarce resource. The scarce
resource in this problem is machine hours. XY-7 BD-4 Sales price $4.00 $3.00 Variable costs $3.00 $2.50
Contribution margin $1.00 $ .50 Mach. hrs. to produce .75 ($ .75/$1.00) .20 ($.20/$1.00) CM per Machine hour
$1.33 ($1.00/ .75) $2.50 ($.50/ .20) Since BD-4 has the highest contribution margin per machine hrs then only
BD-4 should be produced. Total contribution margin would be $250,000 ($2.50 CM X 100,000 machine hrs.) It
is always best to optimize contribution margin of scarce resource.

B. See correct answer.

C. See correct answer.

D. See correct answer.

Question 32 - CMA 687 4-13 - Decision Making

Donnelly Corporation manufactures and sells T-shirts imprinted with college names and slogans. Last year, the shirts
sold for $7.50 each, and the variable cost to manufacture them was $2.25 per unit. The company needed to sell
20,000 shirts to break even. The net income last year was $5,040. Donnelly's expectations for the coming year include
the following:

•The sales price of the T-shirts will be $9


•Variable cost to manufacture will increase by one-third
•Fixed costs will increase by 10%
•The income tax rate of 40% will be unchanged

If Donnelly Corporation wishes to earn $22,500 in net income for the coming year, the company's sales volume in
dollars must be

A. $229,500

(c) HOCK international, page 19


Part 3 : 11/11/10 07:42:55

B. $213,750
C. Some amount other than those given.
D. $257,625

A. This question is a bit more entail because it is asking you to find the sales volume given an after-tax net
income of $22,500. The formula for the problem would be: (Total fixed cost + Before-tax net income) /
contribution margin per unit (%). = $115,500 (fixed costs) + $37,500 before-tax profit ($22,500 / 1- tax rate) /
66.66% contribution margin rate ($6 contribution margin / $9 sales price) = $229,500

B. See correct answer.

C. The correct answer of $229,500 is given.

D. See correct answer.

Question 33 - CMA 1290 4-6 - Decision Making

MultiFrame Company has the following revenue and cost budgets for the two products it sells.

Plastic Frames Glass Frames


Budgeted unit sales 100,000 300,000
Sales price $10.00 $15.00
Direct materials (2.00) (3.00)
Direct labor (3.00) (5.00)
Fixed overhead (3.00) (4.00)
Net income per unit $2.00 $3.00

The budgeted unit sales equal the current unit demand, and total fixed overhead for the year is budgeted at $975,000.
Assume that the company plans to maintain the same proportional mix. In numerical calculations, MultiFrame rounds
to the nearest cent and unit.

The total number of units needed to break even if sales were budgeted at 150,000 units of plastic frames and 300,000
units of glass frames with all other costs remaining constant is

A. 418,455 units.
B. 171,958 units.
C. 365,168 units.
D. 153,948 units.

A. See correct answer.

B. See correct answer.

C. See correct answer.

D.

The breakeven calculation for a multi-product company is a bit more complicated than the breakeven
calculation for a single product company. The first step is to calculate the composite unit contribution
margin.

For 150,000 units of plastic frames and 300,000 units of glass frames, the sales ratio is 1:2. The unit
contribution margin for plastic frames is $10 − $2 DM − $3 DL, or $5. The unit contribution margin for glass
frames is $15 − $3 DM − $5 DL, or $7. Therefore, the composite UCM is ($5 × 1) + ($7 × 2), or $19. Fixed costs

(c) HOCK international, page 20


Part 3 : 11/11/10 07:42:55

are $975,000, so the breakeven point at a ratio of 1:2 is $975,000 ÷ $19, or 51,316 composite units. These will
consist of (51,316 × 1) plastic frames and (51,316 × 2) glass frames, or 102,632 glass frames, or 153,948 units
in total.

Question 34 - HOCK DA 4 - Decision Making

Stark Rehabilitation Hospital is an inpatient institution for severely injured patients who need intense therapy to
recover normal functioning. The hospital has a maximum capacity of 130 patients. The hospital employs nurses,
doctors and physical therapists. Expenses are as follows:

Medical staff:
- 1 nurse for every 20 patients, average cost $66,000 per year per nurse
- 1 doctor for every 60 patients, average cost $150,000 per year per doctor
- 1 physical therapist for every 10 patients, average cost $80,000 per year per therapist

The cost per patient per day for meals, medicine and supplies averages $60 per day per patient.

Utilities cost a base amount of $2,500 per month plus an average of $200 per patient per month.

Medical and exercise equipment, occupancy expense and administrative expense total $7,700,000 per year.

The hospital patient census (i.e., number of patients at any one time) varies from a minimum of 100 patients to fully
occupied at 130 patients.

Stark charges $300 per day per patient, and this includes the room, meals, medicine, supplies and all services.

What are Stark's semi-variable costs?

A. Medical and exercise equipment, occupancy expense and administrative expense


B. Medical staff
C. Meals, medicine and supplies
D. Utilities

A. A semi-variable cost has both a fixed component and a variable component. There is a basic fixed amount that
must be paid regardless of activity; and added to that fixed amount is an amount which varies with activity. An
example might be charges made by a credit card processor, where the processor charges a certain basic monthly fee
whether or not there is any activity, plus a per-transaction charge. Medical and exercise equipment, occupancy
expense and administrative expense do not fit this description.

B. A semi-variable cost has both a fixed component and a variable component. There is a basic fixed amount that
must be paid regardless of activity; and added to that fixed amount is an amount which varies with activity. An
example might be charges made by a credit card processor, where the processor charges a certain basic monthly fee
whether or not there is any activity, plus a per-transaction charge. Medical staff expenses do not fit this description.

C. A semi-variable cost has both a fixed component and a variable component. There is a basic fixed amount that
must be paid regardless of activity; and added to that fixed amount is an amount which varies with activity. An
example might be charges made by a credit card processor, where the processor charges a certain basic monthly fee
whether or not there is any activity, plus a per-transaction charge. Meals, medicine and supplies do not fit this
description.

D. A semi-variable cost has both a fixed component and a variable component. There is a basic fixed amount
that must be paid regardless of activity; and added to that fixed amount is an amount which varies with
activity. An example might be charges made by a credit card processor, where the processor charges a

(c) HOCK international, page 21


Part 3 : 11/11/10 07:42:55

certain basic monthly fee whether or not there is any activity, plus a per-transaction charge. Utilities would fit
this description better than any of the other answer choices. For example, some utility expenses are required
just to maintain the building and the needs of the administrative staff, even if there were no patients. Here,
that basic amount is $2,500 per month. With each additional patient, the cost increases by $200 per month
because each additional patient requires more utilities.

Question 35 - CMA 679 5-26 - Decision Making

Moorehead Manufacturing Company produces two products for which the following data have been tabulated. Fixed
manufacturing cost is applied at a rate of $1.00 per machine hour.

Per Unit XY-7 BD-4


Selling price $4.00 $3.00
Variable manufacturing cost $2.00 $1.50
Fixed manufacturing cost $ .75 $ .20
Variable selling cost $1.00 $1.00

The sales manager has had a $160,000 increase in the budget allotment for advertising and wants to apply the money
to the most profitable product. The products are not substitutes for one another in the eyes of the company's
customers.

Suppose the sales manager chooses to devote the entire $160,000 to increased advertising for BD-4. The minimum
increase in sales dollars of BD-4 required to offset the increased advertising would be

A. $1,600,000
B. $160,000
C. $960,000
D. $320,000

A. See correct answer.

B. See correct answer.

C. The answer to find the minimum increase in dollar sales of BD-4 is to divide the increase in advertising by
the contribution margin percentage. Sales -$3.00 Less: VC -$2.50 CM -$ .50 CM% -16.67% $160,000 increase in
advertising cost =16.67% =$960,000

D. See correct answer.

Question 36 - HOCK DA 3 - Decision Making

Stark Rehabilitation Hospital is an inpatient institution for severely injured patients who need intense therapy to
recover normal functioning. The hospital has a maximum capacity of 130 patients. The hospital employs nurses,
doctors and physical therapists. Expenses are as follows:

Medical staff:
- 1 nurse for every 20 patients, average cost $66,000 per year per nurse
- 1 doctor for every 60 patients, average cost $150,000 per year per doctor
- 1 physical therapist for every 10 patients, average cost $80,000 per year per therapist

The cost per patient per day for meals, medicine and supplies averages $60 per day per patient.

(c) HOCK international, page 22


Part 3 : 11/11/10 07:42:55

Utilities cost a base amount of $2,500 per month plus an average of $200 per patient per month.

Medical and exercise equipment, occupancy expense and administrative expense total $7,700,000 per year.

The hospital patient census (i.e., number of patients at any one time) varies from a minimum of 100 patients to fully
occupied at 130 patients.

Stark charges $300 per day per patient, and this includes the room, meals, medicine, supplies and all services.

What are Stark's semi-fixed costs?

A. Medical staff
B. Medical and exercise equipment, occupancy expense and administrative expense
C. Meals, medicine and supplies
D. Utilities

A.

Semi-fixed costs are fixed over a given range of activity and vary at certain volume increments. A semi-fixed
cost moves upward in a "step" fashion, staying at a certain level over a small range and then moving to the
next level quickly. This description matches the behavior of the medical staff costs.

This is a different concept from that of fixed costs, which we say are fixed as long as the volume remains
within a designated range. For a 100% fixed cost, the relevant range will be much larger than the ranges
within which the medical staff in this example remains the same.

Semi-fixed costs are also referred to sometimes as "semi-variable costs." However, a better term for a
"step-function" cost such as that of the medical staff is "semi-fixed," because true semi-variable costs do not
move upward in steps.

B.

Semi-fixed costs are fixed over a given range of activity and vary at certain volume increments. That does not explain
the behavior of these expenses.

C.

Semi-fixed costs are fixed over a given range of activity and vary at certain volume increments. That does not
describe the behavior of meals, medicine and supplies costs.

D.

Semi-fixed costs are fixed over a given range of activity and vary at certain volume increments. That does not
describe the behavior of the utilities expenses.

Question 37 - CMA 1286 5-13 - Decision Making

The margin of safety is a key concept of CVP analysis. The margin of safety is

A. The difference between budgeted contribution margin and breakeven contribution margin.
B. The difference between budgeted sales and breakeven sales.
C. The difference between the breakeven point in sales and cash flow breakeven.
D. The contribution margin rate.

(c) HOCK international, page 23


Part 3 : 11/11/10 07:42:55

A. The contribution margin is sales less variable costs.

B. The margin of safety is the amount by which sales can decrease before losses can occur (budgeted/actual
sales - sales level at breakeven). This is major concept of CVP analysis.

C. The difference between breakeven point in sales and cash flow breakeven is irrelevant to financial analysis.

D. The contribution margin rate is unit contribution margin divided by unit sales price.

Question 38 - CIA 1188 IV-17 - Decision Making

Which of the following is a characteristic of a contribution income statement?

A. Fixed and variable operating expenses are combined as one line item, but fixed manufacturing expenses are
shown separately from variable manufacturing expenses.
B. Fixed expenses are listed separately from variable expenses.
C. Fixed and variable manufacturing costs are combined as one line item, but fixed operating expenses are shown
separately from variable operating expenses.
D. Fixed and variable expenses are combined as one line.

A. On the contribution income statement, all fixed and variable costs are separated, not just fixed manufacturing costs.

B. The contribution income statement separates out fixed and variable cost on the financial statement. This is
necessary for CVP analysis.

C. On the contribution income statement, all fixed and variable costs are separated, not just fixed operating costs.

D. On the contribution income statement, fixed and variable costs are separated.

Question 39 - CMA 1291 4-14 - Decision Making

Siberian Ski Company recently expanded its manufacturing capacity, which will allow it to produce up to 15,000 pairs
of cross-country skis of the mountaineering model or the touring model. The Sales Department assures management
that it can sell between 9,000 pairs and 13,000 pairs of either product this year. Because the models are very similar,
Siberian Ski will produce only one of the two models.
The following information was compiled by the Accounting Department.

Per Unit (Pair) Data


Mountaineering Touring
Selling price $88.00 $80.00
Variable costs 52.80 52.80

Fixed costs will total $369,600 if the mountaineering model is produced but will be only $316,800 if the touring model
is produced. Siberian Ski is subject to a 40% income tax rate.

The total sales revenue at which Siberian Ski Company would make the same profit or loss regardless of the ski
model it decided to produce is

A. $422,400.
B. $924,000.
C. $686,400.

(c) HOCK international, page 24


Part 3 : 11/11/10 07:42:55

D. $880,000.

A. See correct answer.

B. See correct answer.

C. See correct answer.

D.

We need to know the level of revenue that would be the same for both models at which the profit would also
be the same for both models.

Let M = the quantity of the Mountaineering model and T = the quantity of the Touring model. Since we have
two unknowns, we will need two different equations, both using both unknowns. The question asks for a level
of total revenue which will be the same for both models at which the profit will also be the same for both
models. So one of our equations will need to be a revenue function and the other will need to be a profit
function.

Since the profit must be the same for both models, we will create an expression of the profit for each model,
using the contribution margins for each model multiplied by the quantity for each and from that subtracting
the fixed cost for each. Since the profit must be the same, we set the two expressions equal to one another:

35.2M - 369,600 = 27.2T - 316,800

Since the revenue must also be the same for both models, next we create an expression of the revenue for
each model and then set those two expressions equal to one another:

88M = 80T

We now have two equations in two unknowns. There is more than one way to solve these two equations. Here
is one of the ways.

Simplify the the revenue equation so that M is expressed in terms of T by dividing both sides of the revenue
equation by 88. The result is M = .90909T

Take this value for M expressed in terms of T and plug it into the profit equation in place of M. We now have
an equation in only one variable, T, and we can simplify it and solve for T:

(35.2 * .90909T) - 369,600 = 27.2T - 316,800

32T - 369,600 = 27.2T - 316,800

4.8T = 52,800

T = 11,000

Take this value for T and plug it into the revenue equation in place of T. This will actually give you the answer,
because it will result in total revenue:

88M = (80 ×11,000)

88M = 880,000

However, let's go further and solve for both quantities. Solving for M, we get:

M = 10,000

(c) HOCK international, page 25


Part 3 : 11/11/10 07:42:55

So the quantity of the Touring model is 11,000; and the quantity of the Mountaineering model is 10,000.

Total revenue for the Touring model will be $80 × 11,000, or $880,000. Total revenue for the Mountaineering
model will be $88 ×10,000, or $880,000.

Profit for the Touring model will be ($35.20 × 10,000) - $369,600, or $(17,600). Profit for the Mountaineering
model will be ($27.20 × 11,000) - $316,800, or $(17,600).

Question 40 - CIA 1196 III-91 - Decision Making

A company manufactures a single product. Estimated cost data regarding this product and other information for the
product and the company are as follows:

Sales price per unit $40


Total variable production cost per unit $22
Sales commission (on sales) 5%
Fixed costs and expenses:
Manufacturing overhead $5,598,720
General and administrative $3,732,480
Effective income tax rate 40%

The number of units the company must sell in the coming year in order to reach its breakeven point is

A. 388,800 units.
B. 972,000 units.
C. 583,200 units.
D. 518,400 units.

A. This answer was found by dividing total fixed costs ($9,331,200) by the variable costs ($24).

B. This answer included taxes in the calculation of variable costs. Taxes have to be calculated after fixed costs.

C. The answer for this problem is found by dividing Fixed Cost by unit contribution margin. Sales -$40 Less:
VC (production process) -$22 Less: sales commission (5% of sales) - $ 2 ($40 X 5%) CM -$16 $5,598,720 +
$3,732,480=$16 CM =583,200 units

D. This answer did not include sales commission in the calculation of variable costs.

Question 41 - CMA 1292 4-4 - Decision Making

Richardson Motors uses 10 units of Part No. T305 each month in the production of large diesel engines. The cost to
manufacture one unit of T305 is presented as follows.

Direct materials $ 2,000


Materials handling (20% of direct materials cost) 400
Direct labor 16,000
Manufacturing overhead (150% of direct labor) 24,000
Total manufacturing cost $42,400

Materials handling, which is not included in manufacturing overhead, represents the direct variable costs of the
receiving department that are applied to direct materials and purchased components on the basis of their cost.

(c) HOCK international, page 26


Part 3 : 11/11/10 07:42:55

Richardson's annual manufacturing overhead budget is one-third variable and two-thirds fixed. Simpson Castings, one
of Richardson's reliable vendors, has offered to supply T305 at a unit price of $30,000.

Assume Richardson Motors is able to rent all idle capacity for $50,000 per month. If Richardson decides to purchase
the 10 units from Simpson Castings, Richardson's monthly cost for T305 would

A. Decrease $64,000.
B. Increase $96,000.
C. Decrease $14,000.
D. Increase $46,000.

A.

An answer of $64,000 is the difference between the total cost to manufacture the T305 and the total cost to purchase
the T305. However, it does not consider the fixed manufacturing cost, which will continue whether or not the T305 is
being manufactured. It also does not include the rental income.

B. This answer overlooks the rent of the idle capacity.

C.

An answer of $14,000 does not include the materials handling cost for the purchased components.

D.

Total per unit manufacturing cost for the T305 is $42,400. Since the overhead of $24,000 is 1/3 variable and 2/3
fixed, $16,000 of that would continue if Richardson purchases the T305 outside. Thus, the avoidable variable
manufacturing cost per unit is $26,400.

If Richardson Motors purchases the units from Simpson Castings then the per unit variable cost will be
$36,000 ($30,000 purchase price + $6,000 material handling cost applied [20% handling cost X $30,000 per
unit]. Therefore, by purchasing from Simpson Castings the per unit cost of the T305 component will increase
$9,600 ($36,000 - $26,400). By purchasing 10 units per month, total monthly cost would increase by $96,000
($9,600 X 10). But, if the idle capacity can be rented for $50,000 per month, then the monthly net cost would be
reduced to $46,000 ($96,000 - $50,000).

Question 42 - CMA 686 5-28 - Decision Making

Stewart Industries has been producing two bearings, components B12 and B18, for use in production.

B12 B18
Machine hours required per unit 2.5 3.0
Standard cost per unit:
Direct material $ 2.25 $ 3.75
Direct labor 4.00 4.50
Manufacturing overhead:
Variable (See Note 1) 2.00 2.25
Fixed (See Note 2) 3.75 4.50
$12.00 $15.00

Stewart's annual requirement for these components is 8,000 units of B12 and 11,000 units of B18. Recently, Stewart's
management decided to devote additional machine time to other product lines resulting in only 41,000 machine hours
per year that can be dedicated to the production of the bearings. An outside company has offered to sell Stewart the
annual supply of the bearings at prices of $11.25 for B12 and $13.50 for B18. Stewart wants to schedule the otherwise

(c) HOCK international, page 27


Part 3 : 11/11/10 07:42:55

idle 41,000 machine hours to produce bearings so that the company can minimize its costs (maximize its net benefits).

Note 1. Variable manufacturing overhead is applied on the basis of direct labor hours.

Note 2. Fixed manufacturing overhead is applied on the basis of machine hours.

Assume that Stewart's idle capacity of 41,000 machine hours has a traceable unavoidable annual fixed cost of
$44,000 that will continue if the capacity is not used. The maximum price Stewart would be willing to pay a supplier for
component B18 is

A. $14.00.
B. Some amount other than those given.
C. $10.50.
D. $14.50.

A. See the correct answer.

B.

The first thing needed is to determine which component should receive priority on the limited resource of
machine hours. To do this, we need to calculate the loss per unit of the constrained resource if B12 is
purchased outside versus being manufactured in-house, and the loss per unit of the constrained resource if
B18 is purchased outside versus being manufactured in-house. To do this, we total all of the variable costs
per unit ($2.25 + $4.00 + $2.00 = $8.25 for B12, and $3.75 + $4.50 + $2.25 = $10.50 for B18). We then subtract
the quoted price from the supplier for each of the components from their variable costs to produce in-house.
This give us $8.25 - $11.25, or $(3.00) for B12, and $10.50 - $13.50, or $(3.00) for B18. This gives us the cost
per component to purchase the parts from the outside supplier. Then, we divide each of those total costs per
component by the number of machine hours required to produce each product to calculate the cost per
machine hour, which will be the cost per unit of the constrained resource. This gives us $(3.00) / 2.5, or
$(1.20) for B12 and $(3.00) / 3.0, or $(1.00) for B18.

Since the cost per unit of the constrained resource (machine hours) is higher to purchase B12 outside than it
is to purchase B18 outside ($1.20 for B12 versus $1.00 for B18), Stewart should produce all of the B12 that it
needs in-house and as much B18 as it can produce in the remaining machine time. The amount that it needs
of B18 which it cannot produce itself should be purchased outside. This will minimize the additional cost that
will result from having to purchase some of its components from an outside supplier. This will mean
producing 8,000 units of B12, requiring 2.5 machine hours each for a total of 20,000 hours; and in the 21,000
hours left, producing 21,000 / 3, or 7,000 units of B18; and purchasing the remainder of the B18 units needed.

Since the $44,000 in fixed costs is unavoidable and will continue whether the equipment is used or not, this
fixed cost is not relevant to this decision and should not be included in any of the calculations.

This question asks what is the maximum amount Stewart would be willing to pay an outside supplier for B18.
The alternative to buying some of its requirements of B18 outside is buying some of its requirements of B12
outside. If the additional cost to purchase B18 outside were to increase above the additional cost to purchase
B12 outside, then Stewart would choose to purchase the B12 outside instead. Thus, we need to consider the
additional cost per machine hour that Stewart would have to pay for the B12 to be the upper limit for
additional cost per machine hour to pay for the B18. (Because at a price for B18 that is above that additional
cost, Stewart will purchase the B12 outside instead.)

Since B18 requires 3 hours of machine time to produce one unit, the maximum amount that Stewart would
pay an outside supplier for B18 would include $1.20 * 3 (the loss per machine hour to produce one unit of B12
* the number of hours required to produce one unit of B18). Therefore, the total cost that Stewart would pay
an outside supplier for B18 would be $10.50 (the company's variable cost per unit) plus a $3.60 cost to
purchase B18 outside, for a total of $14.10.

C. This is the total variable cost per unit ($3.75 + $4.50 + $2.25). If the company had idle capacity and could produce

(c) HOCK international, page 28


Part 3 : 11/11/10 07:42:55

all it needed, this would be the most it would pay to an outside supplier. However, Stewart does not have adequate
capacity to produce all that it needs, and so the company must obtain some of its components from an outside
supplier.

D. See the correct answer.

Question 43 - CIA 1196 III-90 - Decision Making

Two companies produce and sell the same product in a competitive industry. Thus, the selling price of the product for
each company is the same. Company 1 has a contribution margin ratio of 40% and fixed costs of $25 million.
Company 2 is more automated, making its fixed costs 40% higher than those of Company 1. Company 2 also has a
contribution margin ratio that is 30% greater than that of Company 1. By comparison, Company 1 will have the _____
breakeven point in terms of dollar sales volume and will have the _____ dollar profit potential once the indifference
point in dollar sales volume is exceeded.

A. Higher, Greater
B. Lower, Greater
C. Higher, Lesser
D. Lower, Lesser

A. Company 1 has lower fixed costs, so it will have a lower breakeven point. Also, Company 1 has a lower contribution
margin, so once the indifference point is exceeded it will also have lower profit.

B. Since Company 1 has a lower contribution margin, it will have lower profit once the indifference point is exceeded.

C. Company 1 has lower fixed costs, so it will have a lower breakeven point.

D. Based on the given information, Company 1 will have a lower breakeven point because its fixed costs are
lower. Also, once the indifference point in dollar sales volume is exceeded, Company 2 will have higher dollar
profit because its contribution margin is 30% greater than Company 1.

Question 44 - CMA 696 4-22 - Decision Making

When a multi-product plant operates at full capacity, quite often decisions must be made as to which products to
emphasize. These decisions are frequently made with a short-run focus. In making such decisions, managers should
select products with the

A. Highest individual unit contribution margin.


B. Highest sales price per unit.
C. Highest sales volume potential.
D. Highest contribution margin per unit of the constraining resource.

A. If operating at full capacity a product may have a higher unit contribution margin, but the product may also require
greater use of limited resources, so its contribution margin per unit of the constraining resource may be lower than
other products.

B. Selecting a product based on the highest sales price is not including the cost of production of the product, so this is
not necessarily maximizing the company's short-term profit.

C. Selecting a product based on the highest sales volume is not including the cost of production of the product, so
therefore, this might not maximize the company's short-term profit.

(c) HOCK international, page 29


Part 3 : 11/11/10 07:42:55

D. In the short-term the best method of measuring profitability is the contribution margin. Since, resources in
the short-term are limited (constrained) the best method to maximize profitability is to select those products
that produce the highest contribution margin per unit of the constraining resource.

Question 45 - CMA 1294 4-2 - Decision Making

United Industries manufactures three products at its highly automated factory. The products are very popular, with
demand far exceeding the company's ability to supply the marketplace. To maximize profit, management should focus
on each product's

A. Contribution margin per machine hour.


B. Contribution margin ratio.
C. Gross margin.
D. Segment margin.

A. The contribution margin per machine hour will indicate which product is the most profitable based on a
scarce resource (machine hours). The company will maximize its profitability by producing those products
that have the greatest contribution margin per scarce resource.

B. The contribution margin ratio is useful only if it is put into sales dollars. It is possible that a higher ratio, lower
volume product could be unprofitable.

C. Since the company can sale all the product it can produce the limiting factor is the use of scarce resources
(machine hours). By maximizing the contribution margin per scarce resource will the company be able to maximize its
profitability.

D. Since the company can sale all the product it can produce the limiting factor is the use of scarce resources
(machine hours). By maximizing the contribution per scarce resource will the company be able to maximize its
profitability.

Question 46 - CIA 1186 IV-16 - Decision Making

The data available for the current year are given below.

Whole company Division 1 Division 2


Variable manufacturing cost of goods sold $400,000 $220,000 $80,000
Unallocated costs (e.g., president's salary) 100,000
Fixed costs controllable by division managers
90,000 50,000 40,000
(e.g., advertising, engineering supervision costs)
Net revenue 1,000,000 600,000 400,000
Variable selling and administrative costs 130,000 70,000 60,000
Fixed costs controllable by others
120,000 70,000 50,000
(e.g., depreciation, insurance)

Based upon the information presented above, the contribution margin for the company was

A. $470,000
B. $530,000
C. $400,000
D. $600,000

(c) HOCK international, page 30


Part 3 : 11/11/10 07:42:55

A. Contribution margin is sales minus variable costs.

B. See correct answer.

C. See correct answer.

D. See correct answer.

Question 47 - CIA 585 IV-6 - Decision Making

Green Company produces Product A and sells it for $18.00. The following cost data apply:

Type of cost Per Unit


Direct materials (3 lb. x $1.50) $4.50
Direct labor 6.45
Variable overhead 1.35
Fixed overhead 1.50
Variable selling expence 1.10
Fixed selling expense 2.20
$17.10

Green has thought of marketing a new Product B with the same cost structure as Product A except that the price will
be $15.60. Green Company currently has the plant capacity necessary for this expansion. Because of the cost
structure, Green Company will find the production and sale of Product B in the short run to be

A. Profitable to produce and sell Product B in the short run at the price of $15.60.
B. Not profitable at $15.60 because the fixed selling expense and fixed manufacturing overhead will not be covered by
the price.
C. Not profitable at any price.
D. Not profitable unless the price can be raised to $17.10.

A. Assuming the company has excess capacity the company will be better off if it is able to sell Product B
above its incremental costs $13.40 ($4.50 DM + $6.45 DL + $1.35 variable overhead + $1.10 variable selling
cost). At the price of $15.60 per unit, Product B would contribute $2.20 per unit toward profit.

B. Fixed selling and fixed manufacturing overhead do not need to be included in price of Product B because they are
already being covered by Product A.

C. As long as the price for Product B is greater than its variable costs it would be profitable in the short-term to
produce and sell Product B.

D. It would be profitable in the short-term to produce and sell Product B at a price greater than its variable costs
$13.40.

Question 48 - CMA 693 4-2 - Decision Making

Delphi Company has developed a new project that will be marketed for the first time during the next fiscal year.
Although the Marketing Department estimates that 35,000 units could be sold at $36 per unit, Delphi's management
has allocated only enough manufacturing capacity to produce a maximum of 25,000 units of the new product annually.
The fixed costs associated with the new product are budgeted at $450,000 for the year, which includes $60,000 for
depreciation on new manufacturing equipment. Data associated with each unit of product are presented as follows.

(c) HOCK international, page 31


Part 3 : 11/11/10 07:42:55

Delphi is subject to a 40% income tax rate.

Variable Costs
Direct material $7.00
Direct labor 3.50
Manufacturing overhead 4.00
Total variable manufacturing cost 14.50
Selling expenses 1.50
Total variable cost $16.00

The maximum after-tax profit that can be earned by Delphi Company from sales of the new product during the next
fiscal year is

A. $30,000.
B. $66,000.
C. $110,000.
D. $50,000.

A. If the breakeven point in sales volume is 22,500 units, this means that any sales volume above this
breakeven point will add to profit. If sales are 25,000 units then 2,500 units will add to profit (25,000 units -
22,500 units, breakeven). Multiply the 2,500 units X $20 contribution margin per unit ($36 sales price - $16
variable cost) = $50,000 gross profit. The question calls for after-tax profit, so after-tax profit will be $30,000
($50,000 X (1 - 40% tax rate).

B. See correct answer.

C. See correct answer.

D. This answer is before-tax profitability.

Question 49 - CMA 686 5-27 - Decision Making

Stewart Industries has been producing two bearings, components B12 and B18, for use in production.

B12 B18
Machine hours required per unit 2.5 3.0
Standard cost per unit:
Direct material $ 2.25 $ 3.75
Direct labor 4.00 4.50
Manufacturing overhead:
Variable (See Note 1) 2.00 2.25
Fixed (See Note 2) 3.75 4.50
$12.00 $15.00

Stewart's annual requirement for these components is 8,000 units of B12 and 11,000 units of B18. Recently, Stewart's
management decided to devote additional machine time to other product lines resulting in only 41,000 machine hours
per year that can be dedicated to the production of the bearings. An outside company has offered to sell Stewart the
annual supply of the bearings at prices of $11.25 for B12 and $13.50 for B18. Stewart wants to schedule the otherwise
idle 41,000 machine hours to produce bearings so that the company can minimize its costs (maximize its net benefits).

Note 1. Variable manufacturing overhead is applied on the basis of direct labor hours.

Note 2. Fixed manufacturing overhead is applied on the basis of machine hours.

(c) HOCK international, page 32


Part 3 : 11/11/10 07:42:55

Stewart will maximize its net benefits by

A. Purchasing 4,800 units of B12 and manufacturing the remaining bearings.


B. Purchasing 8,000 units of B12 and manufacturing 11,000 units of B18.
C. Purchasing 4,000 units of B18 and manufacturing the remaining bearings.
D. Purchasing 11,000 units of B18 and manufacturing 8,000 units of B12.

A. This is not the most cost effective method available to the company. The company will be paying $3.00 more per
unit to manufacture B12 units ($11.25 - $8.25 = $3.00).

B.

This is not the most cost effective method available to the company. Not all available machine hours would be used in
the manufacturing process.

C. By purchasing B12 the company's cost will increase by $3.00 per unit, or $1.20 per machine hr. By
purchasing B18, the cost will increase by $3.00 per unit, or $1.00 per machine hr. Since the loss per machine
hour (the constrained resource) is greater to purchase B12, the company would be better off to manufacture
all 8,000 units of B12. With the constraint of 41,000 machine hours, the company would have 21,000 hours
that would be available to produce B18 (8,000 units of B12 * 2.5 machine hours per unit = 20,000 hours. 41,000
- 20,000 = 21,000 hours remaining). Based on the 21,000 hours of available machine hours left, the company
would be able to produce only 7,000 units of B18 (21,000 ÷ 3 machine hours per unit = 7,000 units). The
remaining units — 11,000 minus 7,000, or 4,000 units — would have to be purchased.

D. After manufacturing 8,000 units of B12 the company would still have enough machine hours available to produce
some B18.

Question 50 - CIA 1184 IV-3 - Decision Making

Two companies are expected to have annual sales of 1,000,000 decks of playing cards next year. Estimates for next
year are presented below:

Company 1 Company 2
Selling price per deck $3.00 $3.00
Cost of paper per deck .62 .65
Printing ink per deck .13 .15
Labor per deck .75 1.25
Variable overhead per deck .30 .35
Fixed costs $960,000 $252,000

Given these data, which of the following responses is correct?

1) Breakeven Point in Units for Company 1


2) Breakeven Point in Units for Company 2
3) Volume in Units at Which Profits of Company 1 and Company 2 are equal

A. 533,334 : 105,000 : 1,180,000


B. 800,000 : 420,000 : 1,180,000
C. 800,000 : 420,000 : 1,000,000
D. 533,334 : 105,000 : 1,000,000

A. See correct answer.

B. Company 1 Company 2 Sales price $3.00 $3.00 Variable costs $1.80 $2.40 Contribution margin $1.20 $0.60

(c) HOCK international, page 33


Part 3 : 11/11/10 07:42:55

Fixed costs $960,000 $252,000 Contribution margin $1.20 $0.60 Breakeven point 800,000 units 420,000 units
Units at which profits are equal = [$1.20X - $960,000 = $0.60X - $252,000] = $0.60X = $708,000 = 1,180,000 units

C. See correct answer.

D. See correct answer.

Question 51 - CIA 1192 IV-18 - Decision Making

A manufacturer has been approached by a new customer who wants to place a one-time order for a component
similar to one that the manufacturer makes for another customer. Existing sales will not be affected by acceptance of
this order. The manufacturer has a policy of setting its targeted selling price at 60% over full manufacturing cost. The
manufacturing costs and the targeted selling price for the existing product are presented as follows.

Direct materials $2.30


Direct labor 3.60
Variable manufacturing overhead (applied at 75% of direct labor cost) 2.70
Fixed manufacturing overhead (applied at 150% of direct labor cost) 5.40
Total manufacturing cost $14.00
Markup (60% of full manufacturing cost) 8.40
Targeted selling price $22.40

The manufacturer has excess capacity to produce the quantity of the component desired by the new customer. The
direct materials used in the component for the new customer would cost the manufacturer $0.25 less than the
component currently being made. The variable selling expenses (packaging and shipping) would be the same, or
$0.90 per unit.

Under these circumstances, the minimum unit price at which the manufacturer would accept the special order is one
exceeding

A. $14.80
B. $9.25
C. $14.00
D. $8.35

A. The correct answer is found by including all of the relevant cost of the proposal [($2.30-$0.25) + $3.60 + $2.70 +
$0.90].

B. The answer of $9.25 is found by adding relevant variable cost of the process [($2.30-$0.25) + $3.60 + $2.70 +
$0.90].

C. The answer $14.00 includes all manufacturing cost (fixed and variable), but does not include the additional selling
and marketing cost which would be incurred. It also does not include new cost of material ($2.30-$0.25).

D. The answer $8.35 does not include the additional selling expense of $0.90.

Question 52 - CIA 1195 IV-76 - Decision Making

In which product-mix pricing strategy is it appropriate for the seller to accept any price that exceeds the storage and
delivery costs for the product?

(c) HOCK international, page 34


Part 3 : 11/11/10 07:42:55

A. By-product pricing.
B. Optional-product pricing.
C. Product-bundle pricing.
D. Captive-product pricing.

A. A by-product is a product of little value that was produced during the production process. Any amount
received above and beyond storage and delivery costs will be accepted.

B. Optional-product pricing is to offer other products along with the main product. It is not likely that the secondary
product would have a cost value of zero, so it must be priced above the storage and delivery costs.

C. An example of product-bundle pricing would be season tickets to the theater. This is done to promote the sell of
tickets. The price of the tickets would be lower than if bought separately, but high enough so the seller still can make a
profit off the sale. Therefore, the price would have to above the costs of storage and delivery.

D. An example of captive-product pricing would be camera and film. It's called captive because the products have to
be used with each other. It is often the case that the seller will make more money off of the captive product (film) then
off the main product (camera). Therefore, the captive product would have to be priced above the costs of storage and
delivery.

Question 53 - CMA 692 4-27 - Decision Making

ABD Realty manages five apartment complexes in a three-state area. Summary income statements for each
apartment complex are shown as follows.

ABD Realty
Summary Income Statements
(in thousands)
One Two Three Four Five
Rental income $1,000 $1,210 $2,347 $1,878 $1,065
Expenses 800 1,300 2,600 2,400 1,300
Profit $ 200 $ (90) $ (253) $ (522) $ (235)

Included in the expenses is $1,200,000 of corporate overhead allocated to the apartment complexes based on rental
income. The apartment complex(es) that ABD should consider selling is (are)

A. Apartment complexes Four and Five.


B. Apartment complexes Two, Three, Four, and Five.
C. Apartment complexes Three, Four, and Five.
D. Apartment complex Four.

A. ABD Realty needs to analysis at the contribution per complex before corporate overhead. This is done to
see which complex is able to cover its own operating expenses. It would be best to allocate this corporate
overhead based on total Rental Income of $7,500. Rental Income Allocation % Total Allocation One $1,000
13.33% $160.0 Two $1,210 16.13% $193.6 Three $2,347 31.29% $375.5 Four $1,878 25.04% $300.5 Five $1,065
14.21% $170.4 Total $7,500 100.00% $1,200.0 The next step is to add back the corporate overhead to see
operating income per complex. Profit + Total Allocation = Operating income One $200 $160.0 $360.0 Two $(90)
$193.6 $103.6 Three $(253) $375.5 $122.5 Four $(522) $300.5 $(221.5) Five $(235) $170.4 $(64.6) Based on the
above information complexes Four and Five are still operating at a loss, so therefore, need to be sold.

B. The solution is to calculate operating income before corporate overhead allocation. Only Four and Five have
operating losses and need to be sold.

C. The solution is to calculate operating income before corporate overhead allocation. Only Four and Five have

(c) HOCK international, page 35


Part 3 : 11/11/10 07:42:55

operating losses and need to be sold. Complex Three has an operating income and should not be sold.

D. The solution is to calculate operating income before corporate overhead allocation. Complex Five also has a
negative operating loss and should be sold.

Question 54 - CMA 1290 4-1 - Decision Making

When used in cost-volume-profit analysis, sensitivity analysis

A. Determines the most profitable mix of products to be sold.


B. Allows the decision maker to introduce probabilities in the evaluation of decision alternatives.
C. Is limited because in cost-volume-profit analysis, costs are not separated into fixed and variable components.
D. Is done through various possible scenarios and computes the impact on profit of various predictions of future
events.

A. A major assumption of CVP analysis is the constant mix of products.

B. The expected value analysis allows the decision maker to introduce probabilities in the evaluation of decision
alternatives, not CVP analysis.

C. Under CVP analysis, fixed and variable costs are separated.

D. Sensitivity analysis allows the decision maker to change the input data and then measure the impact on
profit. In respect to CVP analysis the decision maker, for example, has the opportunity to determine the
sensitivity that fixed costs has on the breakeven point.

Question 55 - CMA 696 4-21 - Decision Making

Costs relevant to a make-or-buy decision include variable labor and variable materials as well as

A. Avoidable fixed costs.


B. Unavoidable fixed costs.
C. Property taxes.
D. Depreciation.

A. Relevant costs are those costs that differ among related choices. Thus, the only costs that are relevant in a
make-or-buy decision would include variable labor, variable materials and avoidable fixed costs.

B. Unavoidable fixed costs will continue regardless of the decision to make-or-buy, so therefore, not relevant.

C. Property taxes are fixed and will continue regardless of the decision to make-or-buy.

D. Depreciation expense is fixed and will continue regardless of the decision to make-or-buy, so therefore, not
relevant.

Question 56 - CMA 691 4-12 - Decision Making

The term relevant cost applies to all the following decision situations except the

(c) HOCK international, page 36


Part 3 : 11/11/10 07:42:55

A. Acceptance of a special order.


B. Replacement of equipment.
C. Determination of a product price.
D. Manufacture or purchase of component parts.

A. Relevant cost are important to the decision whether to accept a special order or not.

B. Relevant cost will be important in the decision of replacing equipment.

C. Relevant cost are incremental (differential) cost that will vary among the possible choices. Relevant cost
are important to see cost differences, not in determining product price.

D. Relevant cost will be important to the decision whether to manufacture or purchase component parts.

Question 57 - CMA 1291 4-15 - Decision Making

Siberian Ski Company recently expanded its manufacturing capacity, which will allow it to produce up to 15,000 pairs
of cross-country skis of the mountaineering model or the touring model. The Sales Department assures management
that it can sell between 9,000 pairs and 13,000 pairs of either product this year. Because the models are very similar,
Siberian Ski will produce only one of the two models.
The following information was compiled by the Accounting Department.

Per Unit (Pair) Data


Mountaineering Touring
Selling price $88.00 $80.00
Variable costs 52.80 52.80

Fixed costs will total $369,600 if the mountaineering model is produced but will be only $316,800 if the touring model
is produced. Siberian Ski is subject to a 40% income tax rate.

If the Siberian Ski Company Sales Department could guarantee the annual sale of 12,000 pairs of either model,
Siberian Ski would

A. Be indifferent as to which model is sold because each model has the same variable cost per unit.
B. Produce 12,000 pairs of mountaineering skis because they have a lower breakeven point.
C. Produce 12,000 pairs of mountaineering skis because they are more profitable.
D. Produce 12,000 pairs of touring skis because they have a lower fixed cost.

A. The mountaineering ski has a higher contribution margin, and even with higher fixed costs it will still be more
profitable then the touring skis.

B. Having a lower breakeven point does not necessarily mean that the mountaineering skis will be more profitable
since the sales level of 12,000 units is above the breakeven point for both skis.

C. The easiest way to find the solution is to prepare simple income statements and do a comparison based on
sales of 12,000 units. Mountain Touring Sales $1,056,000 $960,000 Less: variable costs $633,600 $633,600
Less: fixed costs $369,600 $316,800 Net Profit $52,800 $9,600 Based on the presented information the
Mountain ski would be the most profitable, even though fixed costs are greater.

D. Even though the touring skis have a lower fixed cost, the most profitable option is to produce mountaineering skis.

(c) HOCK international, page 37


Part 3 : 11/11/10 07:42:55

Question 58 - CIA 1183 IV-24 - Decision Making

A company produced the following data (rounded) on its product:

Unit Cost Marginal


Units Produced Fixed Variable Total Cost Revenue
1 100 85 185 85 90
2 50 70 120 55 90
3 33 65 98 55 90
4 25 67 92 73 90
5 20 75 95 107 90

If two units of product were produced and sold, the total contribution margin would be

A. $70
B. $40
C. $25
D. $50

A. This is the unit variable costs. The total contribution margin equals total sales minus total variable costs ($180 -
$140 = $40).

B. The unit contribution margin is units sales - unit variable costs. Unit Sales - $90 Less: VC -$70 Unit CM $20
X 2 units = $40 total contribution margin.

C. The total contribution margin equals total sales minus total variable costs ($180 - $140 = $40).

D. This is fixed costs on a per unit basis. The total contribution margin equals total sales minus total variable costs
($180 - $140 = $40).

Question 59 - CMA 691 4-13 - Decision Making

American Coat Company estimates that 60,000 special zippers will be used in the manufacture of men's jackets
during the next year. Reese Zipper Company has quoted a price of $.60 per zipper. American would prefer to
purchase 5,000 units per month, but Reese is unable to guarantee this delivery schedule. In order to ensure
availability of these zippers, American is considering the purchase of all 60,000 units at the beginning of the year.
Assuming American can invest cash at 8%, the company's opportunity cost of purchasing the 60,000 units at the
beginning of the year is

A. $2,640.
B. $1,500.
C. $1,320.
D. $1,440.

A. The correct answer is $1,320.

B. The correct answer is $1,320.

C. Total cost of the zippers is $36,000 (60,000 X .60). If American Coat company decides to purchase all the
zippers at the beginning of the year then the total additional cash outlay for the company will be $33,000
[$36,000 - (5,000 zippers needed the first month X $0.60)]. If these fund could be invested at 8% then the
earned interest would be $2,640. But, since there is a constant decline in zipper inventory during the year you
need to divide by two to get the average earned interest for the year ($2,640 / 2 = $1,320).

(c) HOCK international, page 38


Part 3 : 11/11/10 07:42:55

D. The correct answer is $1,320.

Question 60 - CMA 1287 5-29 - Decision Making

Leland Manufacturing uses 10 units of Part Number KJ37 each month in the production of radar equipment. The unit
cost to manufacture 1 unit of KJ37 is presented below.

Direct materials $ 1,000


Materials handling (20% of direct material cost) 200
Direct labor 8,000
Manufacturing overhead (150% of direct labor) 12,000
Total manufacturing cost $21,200

Material handling represents the direct variable costs of the Receiving Department that are applied to direct materials
and purchased components on the basis of their cost. This is a separate charge in addition to manufacturing
overhead. Leland's annual manufacturing overhead budget is one-third variable and two-thirds fixed. Scott Supply,
one of Leland's reliable vendors, has offered to supply Part Number KJ37 at a unit price of $15,000.

Assume Leland Manufacturing is able to rent all idle capacity for $25,000 per month. If Leland decides to purchase the
10 units from Scott Supply, Leland's monthly cost for KJ37 would

A. Decrease $7,000.
B. Increase $23,000.
C. Increase $48,000.
D. Change by some amount other than those given.

A. This answer is assuming that there is an increase in unit cost of $1,800.

B. Purchasing the part would increase the cost of Part Number KJ37 by $48,000. If Leland is able to rent the
idle capacity for $25,000 per month then the loss would be $23,000 (-$48,000 + $25,000).

C. This is the total monthly loss incurred by Leland [$21,200 - ($15,000 purchase cost + $3,000 unavoidable handling
cost + $8,000 unavoidable fixed cost)] x 10 units sold.

D. See correct answer.

Question 61 - CMA 690 5-12 - Decision Making

Madengrad Company manufactures a single electronic product called Precisionmix. This unit is a batch-density
monitoring device attached to large industrial mixing machines used in flour, rubber, petroleum, and chemical
manufacturing. Precisionmix sells for $900 per unit. The following variable costs are incurred to produce each
Precisionmix device:

Direct labor $180


Direct materials 240
Factory overhead 105
Total variable production costs 525
Marketing costs 75
Total variable costs $600

Madengrad's income tax rate is 40%, and annual fixed costs are $6,600,000. Except for an operating loss incurred in

(c) HOCK international, page 39


Part 3 : 11/11/10 07:42:55

the year of incorporation, the firm has been profitable over the last 5 years.

If Madengrad Company achieves a sales and production volume of 8,000 units, the annual before-tax income (loss)
will be

A. $1,780,000
B. $(4,200,000)
C. $(2,520,000)
D. $(420,000)

A. See correct answer.

B. The answer is calculated the following way: Sales price (per unit) =$900 Less: variable costs =$600 Per unit
contribution margin =$300 Number of units sold =8,000 Total contribution margin =$2,400,000 Less: Fixed
costs =$6,600,000 EBIT =$(4,200,000)

C. See correct answer.

D. See correct answer.

Question 62 - CMA 679 5-25 - Decision Making

Moorehead Manufacturing Company produces two products for which the following data have been tabulated. Fixed
manufacturing cost is applied at a rate of $1.00 per machine hour.

Per Unit XY-7 BD-4


Selling price $4.00 $3.00
Variable manufacturing cost $2.00 $1.50
Fixed manufacturing cost $ .75 $ .20
Variable selling cost $1.00 $1.00

The sales manager has had a $160,000 increase in the budget allotment for advertising and wants to apply the money
to the most profitable product. The products are not substitutes for one another in the eyes of the company's
customers.

Suppose the sales manager chooses to devote the entire $160,000 to increased advertising for XY-7. The minimum
increase in sales units of XY-7 required to offset the increased advertising is

A. 128,000 units.
B. 80,000 units.
C. 640,000 units.
D. 160,000 units.

A. See correct answer.

B. See correct answer.

C. See correct answer.

D. The unit contribution margin is $1 [$4.00 - ($2.00 variable manufacturing cost + $1.00 variable selling)].
=$160,000 increase in advertising cost $1 contribution margin = 160,000 units

(c) HOCK international, page 40


Part 3 : 11/11/10 07:42:55

Question 63 - CIA 586 IV-9 - Decision Making

A company sells two products, X and Y. The sales mix consists of a composite unit of 2 units of X for every 5 units of
Y (2:5). Fixed costs are $49,500. The unit contribution margins for X and Y are $2.50 and $1.20, respectively.

If the company had a profit of $22,000, the unit sales must have been

A. Product X: 23,800, Product Y: 59,500


B. Product X: 32,500, Product Y: 13,000
C. Product X: 5,000, Product Y: 12,500
D. Product X: 13,000, Product Y: 32,500

A. See correct answer.

B. See correct answer.

C. See correct answer.

D. The formula for this answer is: (Total fixed cost + Profit) / composite contribution margin The composite
contribution margin in this problem is $11 [2($2.50) + 5($1.20)]. = $49,500 + $22,000 / $11 composite
contribution margin. = 6,500 composite units The number of units sold of product X is 13,000 (2 X 6,500
composite units). The number of units sold of product Y is 32,500 (5 X 6,500 composite units).

Question 64 - CMA 1294 4-5 - Decision Making

Austin Manufacturing, which is subject to a 40% income tax rate, had the following operating data for the period just
ended.

Selling price per unit $60


Variable cost per unit 22
Fixed costs 504,000

Management plans to improve the quality of its sole product by (1) replacing a component that costs $3.50 with a
higher-grade unit that costs $5.50, and (2) acquiring a $180,000 packing machine. Austin will depreciate the machine
over a 10-year life with no estimated salvage value by the straight-line method of depreciation. If the company wants
to earn after-tax income of $172,800 in the upcoming period, it must sell

A. 19,300 units.
B. 22,500 units.
C. 21,316 units.
D. 23,800 units.

A. See correct answer.

B. Replacing the component with a higher priced component will add $2 to variable costs. The new variable
costs is $24 per unit. Acquiring the packing machine will add $18,000 to fixed costs ($180,000 / 10-years life).
The pre-tax income is $288,000 ($172,800 /(1 - 40%) Fixed costs + additional depreciation + desired pre-tax
income contribution margin = $504,000 + $18,000 + $288,000 ($60 - $24) = 22,500 units

C. See correct answer.

D. See correct answer.

(c) HOCK international, page 41


Part 3 : 11/11/10 07:42:55

Question 65 - CMA 1292 4-3 - Decision Making

Richardson Motors uses 10 units of Part No. T305 each month in the production of large diesel engines. The cost to
manufacture one unit of T305 is presented as follows.

Direct materials $ 2,000


Materials handling (20% of direct materials cost) 400
Direct labor 16,000
Manufacturing overhead (150% of direct labor) 24,000
Total manufacturing cost $42,400

Materials handling, which is not included in manufacturing overhead, represents the direct variable costs of the
receiving department that are applied to direct materials and purchased components on the basis of their cost.
Richardson's annual manufacturing overhead budget is one-third variable and two-thirds fixed. Simpson Castings, one
of Richardson's reliable vendors, has offered to supply T305 at a unit price of $30,000.

If Richardson Motors purchases the ten T305 units from Simpson Castings, the capacity Richardson used to
manufacture these parts would be idle. Should Richardson decide to purchase the parts from Simpson, the
out-of-pocket cost per unit of T305 would

A. Decrease $12,400.
B. Increase $9,600.
C. Decrease $6,400.
D. Increase $3,600.

A.

This answer includes the fixed manufacturing overhead in the costs that would be avoided if the parts were purchased
outside, and it also fails to include the materials handling cost as a part of the cost of purchasing the parts outside.

B.

Total per unit manufacturing cost for the T305 is $42,400. Since the overhead of $24,000 is 1/3 variable and 2/3
fixed, $16,000 of that would continue. Thus, the avoidable variable manufacturing cost per unit is $26,400.

If Richardson Motors purchases the units from Simpson Castings then the per unit variable cost will be
$36,000 [$30,000 purchase price + $6,000 material handling cost applied [20% X $30,000 per unit]. Therefore,
by purchasing from Simpson Castings the per unit cost of the T305 component will increase $9,600 ($36,000 -
$26,400).

C. This answer includes the fixed manufacturing overhead in the costs that would be avoided if the parts were
purchased outside. However, the fixed manufacturing overhead would continue and so it would be the same whether
the parts are manufactured in-house or purchased outside.

D. This answer does not include the materials handling cost for the parts purchased outside.

Question 66 - CMA 692 4-30 - Decision Making

Barnes Corporation manufactures skateboards and is in the process of preparing next year's budget. The pro forma
income statement for the current year is presented as follows.

Sales $1,500,000
Cost of sales:

(c) HOCK international, page 42


Part 3 : 11/11/10 07:42:55

Direct materials 250,000


Direct labor 150,000
Variable overhead 75,000
Fixed overhead 100,000
Gross profit 925,000
Selling and G&A Variable 200,000
Selling and G&A Fixed 250,000
Operating income 475,000

For the coming year, the management of Barnes Corporation anticipates a 10% increase in sales, a 12% increase in
variable costs, and a $45,000 increase in fixed expenses. The breakeven point for next year will be

A. $474,000.
B. $729,027.
C. $214,018.
D. $862,103.

A. See correct answer.

B. The answer for this problem is found by calculating the new sales, variable costs and fixed costs and then
dividing Fixed Cost by the new contribution margin percentage. Sales -$1,500,000 X 110% = $1,650,000 Less:
VC -$675,000 X 112% = $756,000 CM -$825,000$894,000 CM %55%54.18% $350,000 fixed costs + $45,000
increase in FC =54.18% =$729,027 breakeven point

C. See correct answer.

D. See correct answer.

Question 67 - CMA 1293 4-1 - Decision Making

Copeland, Inc. produces X-547 in a joint manufacturing process. The company is studying whether to sell X-547 at the
split-off point or upgrade the product to become Xylene. The following information has been gathered.

1. Selling price per pound of X-547.


2. Variable manufacturing costs of the upgrade process.
3. Avoidable fixed costs of the upgrade process.
4. Selling price per pound of Xylene.
5. Joint manufacturing costs to produce X-547.

Which of these items should be reviewed when making the upgrade decision?

A. 1, 2, and 4.
B. 1, 2, 3, 4, and 5.
C. 1, 2, 3, and 4.
D. 1, 2, 4, and 5.

A. The answer is correct except it did not include avoidable fixed cost, which are also relevant to the decision whether
to upgrade, or not.

B. This answer includes joint manufacturing cost, which are sunk cost and not relevant to the decision of whether to
upgrade or not.

C. When reviewing the data in order to make a decision whether to upgrade or not, only relevant, or
incremental cost are considered. Relevant (incremental) cost include 1, 2, 3 and 4. Joint manufacturing cost

(c) HOCK international, page 43


Part 3 : 11/11/10 07:42:55

are sunk cost so, therefore, not relevant to the decision.

D. Avoidable fixed cost are relevant to the decision, but sunk cost are not.

Question 68 - CIA 594 III-45 - Decision Making

A company has 7,000 obsolete toys carried in inventory at a manufacturing cost of $6 per unit. If the toys are
reworked for $2 per unit, they could be sold for $3 per unit. If the toys are scrapped, they could be sold for $1.85 per
unit. Which alternative is more desirable (rework or scrap) and what is the total dollar amount of the advantage of that
alternative?

A. Scrap, $47,950.
B. Rework, $8,050.
C. Rework, $36,050.
D. Scrap, $5,950.

A. The advantage is to sell as scrap. The net advantage is $5,950 ($12,950 - $7,000).

B. The advantage is to sell as scrap. The net advantage is $5,950 ($12,950 - $7,000).

C. The advantage is to sell as scrap. The net advantage is $5,950 ($12,950 - $7,000).

D. The manufacturing cost of $6.00 is a sunk cost and not relevant to the decision. The net revenue from the
reworking the obsolete toys is $7,000 [($3.00 - $2.00) X 7,000)]. Revenue if the toys are scrapped is $12,950
($1.85 X 7,000). The dollar advantage for selling as scrap is $5,950 ($12,950 - $7,000).

Question 69 - CMA 687 5-27 - Decision Making

Gleason Co. has two products, a frozen dessert and ready-to-bake breakfast rolls, ready for introduction. However,
plant capacity is limited, and only one product can be introduced at present. Therefore, Gleason has conducted a
market study, at a cost of $26,000, to determine which product will be more profitable. The results of the study follow.

Sales of Desserts Sales of Rolls


at $1.80/unit at $1.20/unit
Volume Probability Volume Probability
250,000 .30 200,000 .20
300,000 .40 250,000 .50
350,000 .20 300,000 .20
400,000 .10 350,000 .10

The costs associated with the two products have been estimated by Gleason's cost accounting department and are
shown as follows.

Dessert Rolls
Ingredients per unit $ .40 $ .25
Direct labor per unit .35 .30
Variable overhead per unit .40 .20
Production tooling* 48,000 25,000
Advertising 30,000 20,000

*Gleason treats production tooling as a current operating expense rather than capitalizing it as a fixed asset.

(c) HOCK international, page 44


Part 3 : 11/11/10 07:42:55

Assuming that Gleason elects to produce the frozen dessert, the profit that would have been earned on the breakfast
rolls is a(n)

A. Deferrable cost.
B. Opportunity cost.
C. Sunk cost.
D. Avoidable cost.

A. Deferrable costs are those cost that can be deferred to some future date.

B. . Opportunity cost is the lost revenue of not selecting the next best alternative. Therefore the lost revenue
of the rolls is an opportunity cost.

C. Sunk costs are cost that have no relevance in management's decision.

D. Avoidable costs are those costs that can be eliminated if a specific activity is discontinued.

Question 70 - CMA 1290 4-5 - Decision Making

MultiFrame Company has the following revenue and cost budgets for the two products it sells.

Plastic Frames Glass Frames


Budgeted unit sales 100,000 300,000
Sales price $10.00 $15.00
Direct materials (2.00) (3.00)
Direct labor (3.00) (5.00)
Fixed overhead (3.00) (4.00)
Net income per unit $2.00 $3.00
The budgeted unit sales equal the current unit demand, and total fixed overhead for the year is budgeted at $975,000.
Assume that the company plans to maintain the same proportional mix. In numerical calculations, MultiFrame rounds
to the nearest cent and unit.

The total number of units needed to break even if the budgeted direct labor costs were $2 for plastic frames instead of
$3 is

A. 154,028 units.
B. 144,444 units.
C. 156,000 units.
D. 146,177 units.

A. See correct answer.

B.

The breakeven calculation for a multi-product company is a bit more complicated than the breakeven
calculation for a single product company. The first step is to calculate the composite unit contribution
margin.

For 100,000 units of plastic frames and 300,000 units of glass frames, the sales ratio is 1:3. The unit
contribution margin for plastic frames is $10 − $2 DM -−$2 DL, or $6. The unit contribution margin for glass
frames is $15 − $3 DM − $5 DL, or $7. Therefore, the composite UCM is ($6 × 1) + ($7 × 3), or $27. Fixed costs

(c) HOCK international, page 45


Part 3 : 11/11/10 07:42:55

are $975,000, so the breakeven point at a ratio of 1:3 is $975,000 ÷ $27, or 36,111 composite units. These will
consist of (36,111 × 1) plastic frames and (36,111 × 3) glass frames, or 108,333 glass frames, or 144,444 units
in total.

C. See correct answer.

D. See correct answer.

Question 71 - CIA 578 IV-18 - Decision Making

An organization's sales revenue is expected to be $72,600, a 10% increase over last year. For the same period, total
fixed costs of $22,000 are expected to be the same as last year. If the number of units sold is expected to increase by
1,100, the marginal revenue per unit will be

A. $46
B. $20
C. $6
D. $4

A. See correct answer.

B. See correct answer.

C. Given that expected sales are to be $72,600, and a 10% increase over last year then last years sales had to
have been $66,000 ($72,600 / 110%). This means that sales increased $6,600 over last year. Thus, if units sold
is expected to increase 1.100 units, dividing $6,600 by 1,100 units will give a marginal revenue per unit of $6
($6,600 / 1,100 units).

D. See correct answer.

Question 72 - CMA 1277 5-1 - Decision Making

The term incremental cost refers to

A. The profit forgone by selecting one choice instead of another.


B. A cost that continues to be incurred in the absence of activity.
C. A cost common to all choices in question and not clearly or feasibly allocable to any of them.
D. The difference in total costs that results from selecting one choice instead of another.

A. This is the definition of opportunity cost.

B. This is the definition of fixed cost.

C. This is the definition of common or joint cost.

D. Incremental (differential) cost are additional costs that result when production, or some other factor is
increased.

(c) HOCK international, page 46


Part 3 : 11/11/10 07:42:55

Question 73 - CMA 694 4-19 - Decision Making

Management accountants are frequently asked to analyze various decision situations, including the following:

I. The cost of a special device that is necessary if a special order is accepted.

II. The cost proposed annually for the plant service for the grounds at corporate headquarters.

III. Joint production costs incurred, to be considered in a sell-at-split versus a process-further decision.

IV. The costs associated with alternative uses of plant space, to be considered in a make/buy decision.

V. The cost of obsolete inventory acquired several years ago, to be considered in a keep-versus-disposal decision.

The costs described in situations I and IV are

A. Sunk costs.
B. Discretionary costs.
C. Prime costs.
D. Relevant costs.

A. Sunk cost are cost that are past cost and not relevant to the decision process.

B. Discretionary cost are incurred cost that have no relationship between input and output (advertising).

C. Prime cost include Direct Material and Direct Labor.

D. Situations I and IV are incremental (differential) cost that are relevant to the decision.

Question 74 - CMA 1288 5-15 - Decision Making

Geary Manufacturing has assembled the data pertaining to two popular products as follows. Past experience has
shown that the fixed manufacturing overhead component included in the cost per machine hour averages $10. Geary
has a policy of filling all sales orders, even if it means purchasing units from outside suppliers.

Blender Electric Mixer


Direct materials $6 $11
Direct labor 4 9
Factory overhead at $16 per hour 16 32
Cost if purchased from an outside supplier 20 38
Annual demand (units) 20,000 28,000

If 50,000 machine hours are available, and Geary Manufacturing desires to follow an optimal strategy, it should

A. Produce 20,000 blenders and 15,000 electric mixers, and purchase all other units as needed.
B. Produce 20,000 blenders and purchase all other units as needed.
C. Produce 25,000 electric mixers and purchase all other units as needed.
D. Purchase all units as needed.

A. In order to find the correct answer you will first need to calculate the necessary machine hrs per unit. The
blender requires 1 machine hr per unit ($16 overhead / $16 per hr). The mixer will require 2 machine hrs per
unit ($32 overhead / $16 per hr). Based on the information that Geary has 50,000 machine hrs available the
company can produce either 50,000 blenders or 25,000 mixers, but some units will have to be purchased. Next

(c) HOCK international, page 47


Part 3 : 11/11/10 07:42:55

it is necessary to take out the fixed portion of overhead, which is $10 per machine hr. The variable portion of
overhead for the blender is $6 ($16 - $10). The variable portion of the mixer is $12 [$32 - (($16 - $10) X 2
machine hr)]. Therefore, total variable cost for the blender is $16 ($6 DM + $4 DL + $6 VOH) and for the mixer
is $32 ($11 DM + $9 DL + $12 VOH). The purchase price of the blender from an outside supplier is $20, so the
cost savings to produce the blender is $4 ($20 purchase price - $16 VC). The cost savings on a per hr basis
would be $4 ($4 cost savings / 1 machine hr per unit). For the mixer the purchase price from an outside
supplier is $38, so the cost savings to produce the mixer is $6 ($38 purchase price - $32 VC). On a per hr
basis the cost savings is $3 ($6 cost savings / 2 machine hrs per unit). In this case, the company will be better
off to produce all the necessary blenders first, since the cost savings is $1 per machine hr ($4 - $3). By
producing all the blenders Geary will have 30,000 machine hrs available to produce mixers (50,000 total
machine hrs - 20,000 hrs needed to produce blenders). Since 30,000 machine hrs are available to produce
blenders only 15,000 blenders would be able to be produced (30,000 hrs / 2 machine hr per unit). So, the
optimal solution is to produce 20,000 blenders, 15,000 mixers and purchase the remaining 13,000 mixers.

B. See correct answer.

C. See correct answer.

D. See correct answer.

Question 75 - CMA 696 4-11 - Decision Making

Listed below are a company's monthly unit costs to manufacture and market a particular product.

Manufacturing costs:
Direct materials $2.00
Direct labor 2.40
Variable indirect 1.60
Fixed indirect 1.00
Marketing costs:
Variable 2.50
Fixed 1.50

The company must decide to continue making the product or buy it from an outside supplier. The supplier has offered
to make the product at the same level of quality that the company can make it. Fixed marketing costs would be
unaffected, but variable marketing costs would be reduced by 30% if the company were to accept the proposal. What
is the maximum amount per unit that the company can pay the supplier without decreasing operating
income?

A. $6.75
B. $5.25
C. $8.50
D. $7.75

A. The maximum amount per unit that the company would pay an outside supplier should include only
avoidable costs. Per unit avoidable costs are $6.75 per unit ($2.00 DM + $2.40 DL + $1.60 variable indirect +
$.75 (30% X $2.50 variable marketing)).

B. See correct answer.

C. See correct answer.

D. This answer assumes that $1.00 of indirect fixed cost is avoidable.

(c) HOCK international, page 48


Part 3 : 11/11/10 07:42:55

Question 76 - CMA 694 4-26 - Decision Making

Condensed monthly operating income data for Korbin Inc. for May follows:

Urban Store Suburban Store Total


Sales $80,000 $120,000 $200,000
Variable costs 32,000 84,000 116,000
Contribution margin 48,000 36,000 84,000
Direct fixed costs 20,000 40,000 60,000
Store segment margin 28,000 (4,000) 24,000
Common fixed cost 4,000 6,000 10,000
Operating income 24,000 (10,000) 14,000

Additional information regarding Korbin's operations follows:


One-fourth of each store's direct fixed costs would continue if either store were closed.
Korbin allocates common fixed costs to each store on the basis of sales dollars.
Management estimates that closing the Suburban Store would result in a 10% decrease in the Urban Store's
sales, while closing the Urban Store would not affect the Suburban Store's sales.
The operating results for May are representative of all months.

Korbin is considering a promotional campaign at the Suburban Store that would not affect the Urban Store. Increasing
annual promotional expense at the Suburban Store by $60,000 in order to increase this store's sales by 10% would
result in a monthly increase (decrease) in Korbin's operating income during the year (rounded) of

A. $7,000
B. $487
C. $(1,400)
D. $(5,000)

A. The correct answer is $(1,400).

B. The correct answer is $(1,400).

C. The monthly cost for the advertising is $5,000 ($60,000 / 12). The advertising will increase sales by 10%, or
$3,600 ($36,000 contribution margin X 1.10). Therefore, the advertising promotion will cost the company and
additional $1,400 ($3,600 increase contribution - $5,000 advertising cost).

D. This is incorrect since it does not include the increased sales from the promotion.

Question 77 - CMA 690 5-13 - Decision Making

Madengrad Company manufactures a single electronic product called Precisionmix. This unit is a batch-density
monitoring device attached to large industrial mixing machines used in flour, rubber, petroleum, and chemical
manufacturing. Precisionmix sells for $900 per unit. The following variable costs are incurred to produce each
Precisionmix device:

Direct labor $180


Direct materials 240
Factory overhead 105
Total variable production costs 525
Marketing costs 75

(c) HOCK international, page 49


Part 3 : 11/11/10 07:42:55

Total variable costs $600

Madengrad's income tax rate is 40%, and annual fixed costs are $6,600,000. Except for an operating loss incurred in
the year of incorporation, the firm has been profitable over the last 5 years.

The annual sales volume required for Madengrad Company to break even is

A. 11,000 units.
B. 22,000 units.
C. 13,888 units.
D. 8,400 units.

A. See correct answer.

B. = 22,000 units

C. See correct answer.

D. See correct answer.

Question 78 - CMA 687 5-23 - Decision Making

Gleason Co. has two products, a frozen dessert and ready-to-bake breakfast rolls, ready for introduction. However,
plant capacity is limited, and only one product can be introduced at present. Therefore, Gleason has conducted a
market study, at a cost of $26,000, to determine which product will be more profitable. The results of the study follow.

Sales of Desserts Sales of Rolls


at $1.80/unit at $1.20/unit
Volume Probability Volume Probability
250,000 .30 200,000 .20
300,000 .40 250,000 .50
350,000 .20 300,000 .20
400,000 .10 350,000 .10

The costs associated with the two products have been estimated by Gleason's cost accounting department and are
shown as follows.

Dessert Rolls
Ingredients per unit $ .40 $ .25
Direct labor per unit .35 .30
Variable overhead per unit .40 .20
Production tooling* 48,000 25,000
Advertising 30,000 20,000

*Gleason treats production tooling as a current operating expense rather than capitalizing it as a fixed asset.

The expected value of Gleason's operating profit directly traceable to the sale of frozen desserts is

A. $198,250.
B. $150,250.
C. Some amount other than those given.
D. $120,250.

A. This is the contribution margin for the frozen desserts ($549,000 - variable cost of $350,750).

(c) HOCK international, page 50


Part 3 : 11/11/10 07:42:55

B. This answer is almost correct, but does not include subtracting advertising.

C. See the correct answer.

D. Using the probabilistic approach the expected sales volume is 305,000 units, or $549,000 (305,000 X $1.80).
Total variable cost are $350,750 [(305,000 X ($.4 + $.35 + $.40)]. Contribution margin is $198,250. Production
tooling and advertising are also traceable to the frozen desserts, so these costs need to be subtracted out.
Operating profit is $120,250 ($198,250 - $78,000).

Question 79 - CIA 1186 IV-10 - Decision Making

A company has just completed the final development of its only product, general recombinant bacteria, which can be
programmed to kill most insects before dying themselves. The product has taken 3 years and $6,000,000 to develop.
The following costs are expected to be incurred on a monthly basis for the normal production level of 1,000,000
pounds of the new product:

Direct materials $300,000


Direct labor 1,250,000
Variable factory overhead 450,000
Fixed factory overhead 2,000,000
Variable selling, general, and adm. expenses 900,000
Fixed selling, general, and adm. expenses 1,500,000
Total: $6,400,000

At a sales price of $5.90 per pound, the sales in pounds necessary to ensure a $3,000,000 profit the first year would
be (to the nearest thousand pounds)

A. 25,600,000 pounds.
B. 15,000,000 pounds.
C. 14,000,000 pounds.
D. 13,017,000 pounds.

A. See correct answer.

B. This data information is on a monthly basis. The unit contribution margin is $3 per lb [$5.90 - ($.30 DM +
$1.25 DL + $.45 variable overhead + $.90 variable selling)]. =$1,500,000 fixed selling + 2,000,000 fixed factory +
$250,000 profit (monthly basis)$3 per lb. = 1,250,000 lbs per month X 12 month = 15,000,000 lbs.

C. See correct answer.

D. See correct answer.

Question 80 - CIA 595 IV-74 - Decision Making

Which of the following pricing policies involves the selling company setting freight charges to customers at the actual
average freight cost?

A. Freight absorption pricing.


B. FOB - origin pricing.
C. Zone pricing.

(c) HOCK international, page 51


Part 3 : 11/11/10 07:42:55

D. Uniform delivered pricing.

A. The seller using freight absorption pricing system absorbs all or part of the freight charges.

B. The customer using FOB-origin pricing system pays for actual freight charges.

C. Freight charges using zone pricing is based on location, not on actual average freight cost.

D. Companies like using the uniform delivered pricing system because all customers are charged the same
regardless of their location. Therefore, freight charges are based on actual average freight cost.

Question 81 - CMA 691 4-14 - Decision Making

Regis Company manufactures plugs used in its manufacturing cycle at a cost of $36 per unit that includes $8 of fixed
overhead. Regis needs 30,000 of these plugs annually, and Orlan Company has offered to sell these units to Regis at
$33 per unit. If Regis decides to purchase the plugs, $60,000 of the annual fixed overhead applied will be eliminated,
and the company may be able to rent the facility previously used for manufacturing the plugs.

If Regis Company purchases the plugs but does not rent the unused facility, the company would

A. Save $2.00 per unit.


B. Save $3.00 per unit.
C. Lose $3.00 per unit.
D. Lose $6.00 per unit.

A. This is the fixed cost that will be avoidable by purchasing ($60,000/30,000).

B. This answer is found by subtracting total cost from the suppliers cost ($36.00 - $33.00).

C. The variable cost of plugs is $28.00 per unit ($36.00 - $8.00). This should be the most Regis would be
willing to pay to an outside supplier. But, the question states that $60,000 in annual fixed overhead would be
saved, or $2.00 per unit ($60,000 / 30,000 units). Taking this into account, the most that Regis would then be
willing to then pay is $30.00 per unit ($28.00 + $2). Purchasing the units from Orlan, Regis would lose $3.00
per unit ($30.00 - $33.00).

D. This answer is the total fixed cost that are unavoidable [$8.00 - ($60,000/30,000)].

Question 82 - CMA 1294 4-3 - Decision Making

Marston Enterprises sells three chemicals: petrol, septine, and tridol. Petrol is the company's most profitable product;
tridol is the least profitable. Which one of the following events will definitely decrease the firm's overall breakeven point
for the upcoming accounting period?

A. An increase in the overall market for septine.


B. An increase in anticipated sales of petrol relative to sales of septine and tridol.
C. A decrease in tridol's selling price.
D. The installation of new computer-controlled machinery and subsequent layoff of assembly-line workers.

A. Unclear how an increase in the overall market of septine would effect the breakeven point. The breakeven point
depends on two factors: fixed costs, and unit contribution margin.

B. Petrol is the most profitable product for the firm, therefore increasing the sales of petrol would increase

(c) HOCK international, page 52


Part 3 : 11/11/10 07:42:55

the average unit contribution margin of the three products, thereby decreasing the breakeven point (Fixed
costs/average unit contribution margin).

C. Tridol is the least profitable product, therefore decreasing the selling price would decrease the per unit contribution
margin, thereby increasing the firm's overall breakeven point.

D. Would still be unclear how the new computer-controlled machinery would have on efficiency. Though, the purchase
would definitely increase fixed cost, so in fact the breakeven point may increase.

Question 83 - CMA 1290 4-2 - Decision Making

One of the major assumptions limiting the reliability of breakeven analysis is that

A. Efficiency and productivity will continually increase.


B. Total fixed costs will remain unchanged over the relevant range.
C. Total variable costs will remain unchanged over the relevant range.
D. The cost of production factors varies with changes in technology.

A. It is assumed that efficiency and productivity will remain the same.

B. In order for CVP to actually work mathematically, a number of assumptions need to be made in order to
simplify the real world and all of the variances that occur in it. The main assumptions of CVP are: costs and
revenues are predictable and linear; fixed cost remain constant; total variable cots change in proportion to
actual level; unit variable costs remain constant; inventory does not change; the time value of money is
ignored; prices remain fixed; and all costs are either variable or fixed.

C. A limiting factor is that per unit variable cost will remain the same, not total variable costs.

D. It is assumed that cost of production is stable.

Question 84 - CMA 690 5-16 - Decision Making

Madengrad Company manufactures a single electronic product called Precisionmix. This unit is a batch-density
monitoring device attached to large industrial mixing machines used in flour, rubber, petroleum, and chemical
manufacturing. Precisionmix sells for $900 per unit. The following variable costs are incurred to produce each
Precisionmix device:

Direct labor $180


Direct materials 240
Factory overhead 105
Total variable production costs 525
Marketing costs 75
Total variable costs $600

Madengrad's income tax rate is 40%, and annual fixed costs are $6,600,000. Except for an operating loss incurred in
the year of incorporation, the firm has been profitable over the last 5 years.

Assume a 10% increase in annual fixed costs, a 20% unit cost increase for direct labor, and a reduction in unit
material costs of 25%, with no change in selling price. Madengrad Company's breakeven point would increase
(decrease) (rounded to the nearest whole unit) by

(c) HOCK international, page 53


Part 3 : 11/11/10 07:42:55

A. (1,620) units.
B. 3,960 units.
C. 407 units.
D. 1,604 units.

A. See correct answer.

B. See correct answer.

C. = 407 incremental increase in breakeven point

D. See correct answer.

Question 85 - CIA 1189 IV-25 - Decision Making

Which costs are relevant to the decision to further process a product beyond its current state?

A. Joint costs.
B. Absorption costs.
C. Fixed factory overhead.
D. Incremental costs.

A. Joint cost are not relevant to the decision.

B. Absorption costing includes all manufacturing cost (variable and fixed). Not all manufacturing cost will be relevant to
the decision whether to process beyond its current state.

C. Fixed manufacturing cost are included in absorption costing and may not be relevant to the decision of processing
beyond its current state.

D. When deciding to process further or not, need to look only at additional cost that results when production,
or some other factor is increased.

Question 86 - CIA 583 IV-5 - Decision Making

A company allocates its variable factory overhead based on direct labor hours. During the past 3 months, the actual
direct labor hours and the total factory overhead allocated were as follows:

January February March


Direct labor hours 1,000 3,000 5,000
Total factory overhead allocated $80,000 $140,000 $200,000

Based upon this information, monthly fixed factory overhead was

A. $33,333
B. $50,000
C. $30,000
D. $46,667

A. See correct answer.

(c) HOCK international, page 54


Part 3 : 11/11/10 07:42:55

B. The best method to separate variable and fixed factory overhead costs is the Hi-Low method. First,
calculate the variable portion of the cost. This is done by dividing the difference in total factory overhead
costs by the difference in the direct labor hours. Per unit direct labor costs $30 [($200,000 - $80,000) / (5,000
units - 1,000 units)]. Therefore, assuming that fixed costs remain constant, then total fixed factory overhead
must be $50,000 [$80,000 - (1,000 units X $30)].

C. See correct answer.

D. See correct answer.

Question 87 - CMA 1296 4-22 - Decision Making

In a make-versus-buy decision, the relevant costs include variable manufacturing costs as well as

A. Depreciation costs.
B. Factory management costs.
C. Avoidable fixed costs.
D. General office costs.

A. Depreciation costs will generally not differ among the options, therefore is not relevant.

B. Management costs will generally not differ among the options, therefore is not relevant.

C. In the decision to make-versus-buy decision you want to look only at relevant cost that differ among the
options. Relevant cost include both avoidable fixed and variable costs.

D. General office costs will generally not differ among the options, therefore is not relevant.

Question 88 - CMA 686 5-26 - Decision Making

Stewart Industries has been producing two bearings, components B12 and B18, for use in production.

B12 B18
Machine hours required per unit 2.5 3.0
Standard cost per unit:
Direct material $ 2.25 $ 3.75
Direct labor 4.00 4.50
Manufacturing overhead:
Variable (See Note 1) 2.00 2.25
Fixed (See Note 2) 3.75 4.50
$12.00 $15.00

Stewart's annual requirement for these components is 8,000 units of B12 and 11,000 units of B18. Recently, Stewart's
management decided to devote additional machine time to other product lines resulting in only 41,000 machine hours
per year that can be dedicated to the production of the bearings. An outside company has offered to sell Stewart the
annual supply of the bearings at prices of $11.25 for B12 and $13.50 for B18. Stewart wants to schedule the otherwise
idle 41,000 machine hours to produce bearings so that the company can minimize its costs (maximize its net benefits).

Note 1. Variable manufacturing overhead is applied on the basis of direct labor hours.

Note 2. Fixed manufacturing overhead is applied on the basis of machine hours.

(c) HOCK international, page 55


Part 3 : 11/11/10 07:42:55

The net benefit (loss) per machine hour that would result if Stewart accepts the supplier's offer of $13.50 per unit for
Component B18 is

A. $(1.00)
B. $.50.
C. Some amount other than those given.
D. $(1.75)

A.

Whenever a resource is constrained, as the machine hours are in this problem, we need to work with cost or
contribution margin per unit of the constrained resource, whichever is appropriate. Here, the "cost" is the
amount that the outside supplier will charge over and above the total variable cost to produce the component
in-house. And the "per unit of the constrained resource" is per machine hours. For this problem, we need to
find the net benefit or cost per machine hour for B18 if the units are purchased from the outside supplier.

The first step is to calculate the total variable cost for B18. That is $3.75 for direct materials + $4.50 for direct
labor + $2.25 for variable manufacturing overhead, for a total variable cost of $10.50.

The second step is to determine the difference between the outside company's quote and the total variable
cost and whether it is a benefit or a cost. The outside company's quote is $13.50, and that is $3.00 greater
than the total variable cost of $10.50. So the total cost per unit to buy from the outside supplier is $3.00 per
unit.

The final step is to determine the total cost per unit of the constrained resource, which is machine hours. To
manufacture one unit of B18, it requires 3.0 machine hours. So we divide the total cost per unit to buy from
the outside supplier ($3.00) by the number of machine hours required to manufacture one unit (3.0 machine
hours), and we have the cost per machine hour to purchase the units outside, which is $(1.00).

B. This answer results from comparing the total cost (variable and fixed) of $15.00 per unit with the quote of $13.50
per unit and then dividing by 3 machine hrs per unit. $15.00 - $13.50 = $1.50 / 3 machine hrs. per unit = $.50 per unit
per machine hrs. However, this is not the correct way to find the net benefit or loss per machine hour that would result
if Stewart accepts the supplier's offer of $13.50 per unit instead of manufacturing the components itself.

C. The correct answer is one of those given.

D. This answer is a result of totalling direct material cost ($3.75) and direct labor cost ($4.50) per unit produced,
subtracting the quote of $13.50 from the total ($8.25), and then dividing by the number of machine hours required per
unit (3.0). However, variable manufacturing overhead should be included in the calculation.

Question 89 - CMA 1287 5-30 - Decision Making

Leland Manufacturing uses 10 units of Part Number KJ37 each month in the production of radar equipment. The unit
cost to manufacture 1 unit of KJ37 is presented below.

Direct materials $ 1,000


Materials handling (20% of direct material cost) 200
Direct labor 8,000
Manufacturing overhead (150% of direct labor) 12,000
Total manufacturing cost $21,200

Material handling represents the direct variable costs of the Receiving Department that are applied to direct materials

(c) HOCK international, page 56


Part 3 : 11/11/10 07:42:55

and purchased components on the basis of their cost. This is a separate charge in addition to manufacturing
overhead. Leland's annual manufacturing overhead budget is one-third variable and two-thirds fixed. Scott Supply,
one of Leland's reliable vendors, has offered to supply Part Number KJ37 at a unit price of $15,000.

Assume that Leland Manufacturing does not wish to commit to a rental agreement but could use idle capacity to
manufacture another product that would contribute $52,000 per month. If Leland elects to manufacture KJ37 in order
to maintain quality control, Leland's opportunity cost is

A. $4,000.
B. Some amount other than those given.
C. $18,000.
D. $(20,000).

A. Purchasing the part would increase the total cost of Part Number KJ37 by $48,000 ($4,800 X 10 units). If
Leland has the opportunity to produce another product that would contribute $52,000 per month than the
opportunity cost would be $4,000 ($52,000 - $48,000).

B. See correct answer.

C. The opportunity cost is the lost benefit for not manufacturing another product. Lost benefit is calculated by
subtracting the potential contribution by the additional cost of purchasing the product ($52,000 - $48,000).

D. The opportunity cost is the lost benefit for not manufacturing another product. Lost benefit is calculated by
subtracting the potential contribution by the additional cost of purchasing the product ($52,000 - $48,000).

Question 90 - CMA Sample Q4-2 - Decision Making

All of the following costs are relevant to a decision to accept or reject an order except

A. Out-of-pocket costs.
B. Replacement costs.
C. Differential costs.
D. Sunk costs.

A. Out-of-pocket costs are actual cash outflows that have to be incurred and are relevant to the decision making
process.

B. Replacement cost is the cost to replace an existing asset. It is relevant to the decision making process.

C. Differential (incremental) costs are relevant to the decision whether to accept or reject. These cost are the cost
differences between two options.

D. Sunk cost will not vary with the related options, therefore are irrelevant.

Question 91 - CMA 1294 4-4 - Decision Making

The following information relates to Clyde Corporation, which produced and sold 50,000 units during a recent
accounting period.

Sales $850,000
Manufacturing costs:

(c) HOCK international, page 57


Part 3 : 11/11/10 07:42:55

Fixed 210,000
Variable 140,000
Selling and administrative costs:
Fixed 300,000
Variable 45,000
Income tax rate: 40%

For the next accounting period, if production and sales are expected to be 40,000 units, the company should
anticipate a contribution margin per unit of

A. $1.86.
B. $13.30.
C. $7.30.
D. $3.10.

A. See correct answer.

B. The contribution margin is the difference between sales and variable costs (sales - variable costs). To find
on a per unit basis, need to be divide by 50,000 units sold. Sales -$17 ($850,000 / 50,000 units) Variable costs -
$3.7 ($185,000 / 50,000 units) CM -$13.3

C. See correct answer.

D. See correct answer.

Question 92 - CIA 594 III-43 - Decision Making

The following data pertains to XYZ Company for the current year of operations.

Total Cost Unit Cost


Sales (40,000 units) $1,000,000 $25
Raw materials 160,000 4
Direct labor 280,000 7
Factory overhead:
Variable 80,000 2
Fixed 360,000
Selling and general expenses:
Variable 120,000 3
Fixed 225,000

Assuming that XYZ Company sells 80,000 units, what is the maximum that can be paid for an advertising campaign
while still breaking even?

A. $135,000
B. $1,015,000
C. $535,000
D. $695,000

A. The breakeven volume is 65,000 units [($585,000 FC / $9 CM ($25 - $16)]. Given a sales volume of 80,000
units, the company would produce a before-tax profit of $135,000 [(80,000 units − 65,000) × $9 CM], which
would be the most that company XYZ could pay for advertising and still breakeven.

B. See correct answer.

(c) HOCK international, page 58


Part 3 : 11/11/10 07:42:55

C. See correct answer.

D. See correct answer.

Question 93 - CIA 593 IV-19 - Decision Making

The ABC Company manufactures components for use in producing one of its finished products. When 12,000 units
are produced, the full cost per unit is $35, separated as follows:

Direct materials $5
Direct labor $15
Variable overhead $10
Fixed overhead $5

The XYZ Company has offered to sell 12,000 components to ABC for $37 each. If ABC accepts the offer, some of the
facilities currently being used to manufacture the components can be rented as warehouse space for $40,000.
However, $3 of the fixed overhead currently applied to each component would have to be covered by ABC's other
products. What is the differential cost to the ABC Company of purchasing the components from the XYZ Company?

A. $44,000
B. $24,000
C. $20,000
D. $8,000

A. This answer ignores the $2.00 of unavoidable fixed overhead cost.

B. This answer is incorrect because it is simply taking cost differential between the full manufacturing cost and cost to
purchase, ignoring contribution from rent of warehouse and unavoidable fixed overhead cost.

C. The differential (incremental) cost are those cost differing between the two options. The total cost of
purchasing the component and renting the warehouse space is $404,000 [($37.00 X 12,000) - $40,000]. The
total cost to manufacture is $384,000 [($30.00 variable cost + $2.00 avoidable fixed cost) X 12,000 units]. Total
cost differential is $20,000 ($404,000 - $384,000).

D. The answer of $8.00 is assuming that the $3.00 of fixed overhead that is currently applied to each component is
avoidable.

Question 94 - CIA 594 III-99 - Decision Making

Total production costs of prior periods for a company are listed as follows. Assume that the same cost behavior
patterns can be extended linearly over the range of 3,000 to 35,000 units and that the cost driver for each cost is the
number of units produced.

Production (units/month): 3,000 9,000 16,000 35,000


Cost X: $23,700 $52,680 $86,490 $178,260
Cost Y: 47,280 141,840 252,160 551,600

What is the average cost per unit at a production level of 8,000 units for cost X?

A. $4.83
B. $7.90

(c) HOCK international, page 59


Part 3 : 11/11/10 07:42:55

C. $5.98
D. $5.85

A. This answer is the per unit variable cost. The question is asking of average costs per unit, which includes both
variable and fixed.

B. See correct answer.

C. The answer is asking for average cost per unit, which includes both fixed and variable costs. Based on the
given information, the Hi-Low is the best method to separate these costs. The per unit variable costs is found
by dividing the difference in total costs by the difference in the production volumes. Per unit variable costs is
$4.83 [($178,260 - $23,700) / (35,000 units - 3,000 units)]. Therefore, assuming that fixed costs remain constant
in the production range then total fixed costs must be $9,210 [$23,700 (total cost of X at a production level of
3,000) - (3,000 units X $4.83)]. So, at a production level of 8,000 units, the variable costs would be $38,640
(8,000 units X $4.83). Total production costs are $47,850 ($38,640 + $9,210) and the per unit cost would is
$5.98 ($47,850 / 8,000 units).

D. See correct answer.

Question 95 - CMA 687 4-11 - Decision Making

Donnelly Corporation manufactures and sells T-shirts imprinted with college names and slogans. Last year, the shirts
sold for $7.50 each, and the variable cost to manufacture them was $2.25 per unit. The company needed to sell
20,000 shirts to break even. The net income last year was $5,040. Donnelly's expectations for the coming year include
the following:

•The sales price of the T-shirts will be $9


•Variable cost to manufacture will increase by one-third
•Fixed costs will increase by 10%
•The income tax rate of 40% will be unchanged

The number of T-shirts Donnelly Corporation must sell to break even in the coming year is

A. 17,500
B. 20,000
C. Some amount other than those given.
D. 19,250

A. See correct answer.

B. This is last year's breakeven point.

C. The correct answer of 19,250 units to breakeven is given.

D. In this problem, first need to calculate fixed cost for last year. Given that breakeven volume last year was
20,000 T-shirts, and contribution margin is $5.25 ($7.50-$2.25 variable cost), last years fixed cost must have
been $105,000. Fixed cost are expected to increase by 10%, so the fixed costs in the coming year are
expected to be $115,500 ($105,000 X 110%). The formula to calculate the breakeven point is: Total fixed cost /
contribution margin per unit = $115,500 / $6 contribution margin per unit ($9 sales price - $3 variable costs) =
19,250 units

Question 96 - CIA 1186 IV-17 - Decision Making

(c) HOCK international, page 60


Part 3 : 11/11/10 07:42:55

The data available for the current year are given below.

Whole company Division 1 Division 2


Variable manufacturing cost of goods sold $400,000 $220,000 $80,000
Unallocated costs (e.g., president's salary) 100,000
Fixed costs controllable by division managers
90,000 50,000 40,000
(e.g., advertising, engineering supervision costs)
Net revenue 1,000,000 600,000 400,000
Variable selling and administrative costs 130,000 70,000 60,000
Fixed costs controllable by others
120,000 70,000 50,000
(e.g., depreciation, insurance)

Using the information presented above, the contribution by Division 1 was

A. $260,000
B. $380,000
C. $190,000
D. $310,000

A. This answer is the contribution controllable by the divisional manager, but total contribution is found by subtracting
out the fixed allocated costs (controllable by others) as well.

B. This answer only subtracted variable manufacturing costs from sales revenue, but we need to subtract out all
allocated divisional costs.

C. This question is asking for contribution by division 1, so we need to subtract sales revenue from all costs
allocated to the division.

D. This is the contribution margin, but the question is asking for total contribution by the division, so we need to
subtract out all costs allocated to division.

Question 97 - CIA 1183 IV-23 - Decision Making

A company produced the following data (rounded) on its product:

Unit Cost Marginal


Units Produced Fixed Variable Total Cost Revenue
1 100 85 185 85 90
2 50 70 120 55 90
3 33 65 98 55 90
4 25 67 92 73 90
5 20 75 95 107 90

How many units should be produced?

A. 4
B. 3
C. 2
D. 5

A. Should keep producing until MC = MR. In this case, marginal revenue is greater than its marginal cost at
the production level of 4 units, but at 5 units the marginal cost is greater than its marginal revenue.

(c) HOCK international, page 61


Part 3 : 11/11/10 07:42:55

B. The optimal solution is to produce at 4 units. Production should be increased until MR = MC.

C. The most optimal solution is to produce at 4 units. Production should be increased until MR = MC.

D. Producing 5 units, MC > MR. This is not the optimal solution, since the company would lose income by producing
the one additional unit.

Question 98 - CMA 679 5-13 - Decision Making

BE&H Manufacturing is considering dropping a product line. It currently produces a multi-purpose woodworking clamp
in a simple manufacturing process that uses special equipment. Variable costs amount to $6.00 per unit. Fixed
overhead costs, exclusive of depreciation, have been allocated to this product at a rate of $3.50 a unit and will
continue whether or not production ceases. Depreciation on the special equipment amounts to $20,000 a year. If
production of the clamp is stopped, the special equipment can be sold for $18,000; if production continues, however,
the equipment will be useless for further production at the end of 1 year and will have no salvage value. The clamp
has a selling price of $10 a unit. Ignoring tax effects, the minimum number of units that would have to be sold in the
current year to break even on a cash flow basis is

A. 20,000
B. 5,000
C. 36,000
D. 4,500

A. See correct answer.

B. See correct answer.

C. See correct answer.

D. The question is asking for the breakeven on the cash basis. This being the case, it is necessary to use the
cash value of the equipment that it can be sold for, or $18,000. The rest of the problem is the same. $18,000
(cash value of equipment) =$4 CM ($10 - $6) =4,500 units

Question 99 - CIA 1193 IV-11 - Decision Making

A retail company determines its selling price by marking up variable costs 60%. In addition, the company uses
frequent selling price markdowns to stimulate sales. If the markdowns average 10%, what is the company's
contribution margin ratio?

A. 27.5%
B. 37.5%
C. 30.6%
D. 41.7%

A. See correct answer.

B. See correct answer.

C. The contribution margin is equal to selling price minus variable costs (Sales - VC = CM). If variable cost are
$100 per unit, then sales price is $160 per unit ($100 X 160%). If there is a markdown of 10%, then the new
sales price will $144 ($160 X 90%) and the new contribution margin will be $44 ($144 - $100). The new
contribution margin ratio will be 30.6% ($44 / $144).

(c) HOCK international, page 62


Part 3 : 11/11/10 07:42:55

D. See correct answer.

Question 100 - CMA 691 4-8 - Decision Making

In a manufacturing environment, the best short-term profit maximizing approach would be to

A. Minimize fixed overhead cost per unit by producing at full capacity.


B. Maximize contribution per unit times the number of units sold.
C. Minimize variable costs per unit times the number of units produced.
D. Maximize unit gross profit times the number of units sold.

A. In the short-term fixed cost are not relevant, so therefore not considered.

B. Maximizing its contribution margin (unit sales price - unit variable cost) a company would have enough
contribution to cover fixed cost and contribute to profit.

C. Minimizing variable cost does not consider the price of the product, so therefore might not maximize short-term
profit.

D. This would be a long-term strategy, since gross profit would include any fixed cost. Short-term strategy is to
maximize the company's contribution margin.

Question 101 - CMA 687 4-14 - Decision Making

For a profitable company, the amount by which sales can decline before losses occur is known as the

A. Sales volume variance.


B. Marginal income rate.
C. Hurdle rate.
D. Margin of safety.

A. The Sales volume variance measures the impact of differences in sales volume. The calculation is [(Actual Sales
volume - Budgeted Sales volume) X Standard contribution per unit].

B. The marginal income rate is the rate of return that is gained from making one more sales or one more investment.

C. The hurdle rate is the rate of return that must received on a specific project before it will become acceptable by
management.

D. The margin of safety is the amount by which sales can decrease before losses can occur (budgeted/actual
sales - sales level at breakeven).

Question 102 - CMA 691 4-7 - Decision Making

Total unit costs are

A. Irrelevant in marginal analysis.


B. Needed for determining sunk costs.

(c) HOCK international, page 63


Part 3 : 11/11/10 07:42:55

C. Independent of the cost system used to generate them.


D. Relevant for cost-volume-profit analysis.

A. Marginal analysis (incremental or differential) only look at those cost which differ among the possible
choices. Total unit cost includes all cost (fixed and variable), which some may be irrelevant to the decision
process.

B. Sunk cost are irrelevant to the decision process.

C. Total unit cost is dependent on the cost system that was used to generate them.

D. Total unit cost include both fixed and variable cost on a per unit basis. CVP analysis separates out these cost.

Question 103 - CMA 1288 5-12 - Decision Making

Management accountants are frequently asked to analyze various decision situations including the following.

1. Alternative uses of plant space, to be considered in a make/buy decision.

2. Joint production costs incurred, to be considered in a sell-at-split versus a process-further decision.

3. Research and development costs incurred in prior months, to be considered in a product-introduction decision.

4. The cost of a special device that is necessary if a special order is accepted.

5. The cost of obsolete inventory acquired several years ago, to be considered in a keep-versus-disposal decision.

The costs described in situations 1 and 4 are

A. Prime costs.
B. Discretionary costs.
C. Sunk costs.
D. Relevant costs.

A. Prime costs include only direct materials and direct labor.

B. Discretionary costs are those cost that have no direct input/output relationship.

C. Sunk costs are those cost that have already been incurred and will not be part of management's future decisions.

D. The costs that are best describe in situations 1 and 4 are relevant cost. These types of costs would be part
of the decision to make / buy, or whether to accept the special order.

Question 104 - CMA 687 4-10 - Decision Making

Donnelly Corporation manufactures and sells T-shirts imprinted with college names and slogans. Last year, the shirts
sold for $7.50 each, and the variable cost to manufacture them was $2.25 per unit. The company needed to sell
20,000 shirts to break even. The net income last year was $5,040. Donnelly's expectations for the coming year include
the following:

•The sales price of the T-shirts will be $9


•Variable cost to manufacture will increase by one-third

(c) HOCK international, page 64


Part 3 : 11/11/10 07:42:55

•Fixed costs will increase by 10%


•The income tax rate of 40% will be unchanged

The selling price that would maintain the same contribution margin rate as last year is

A. $9.00
B. Some amount other than those given.
C. $10.00
D. $8.25

A. $9 is the new planned sales price of the T-shirts.

B. To maintain the same contribution rate the new selling price will need to be $10.

C. Last years contribution margin rate is 70% ($5.25 contribution margin ($7.50-$2.25 variable cost)/ $7.50 sale
price). If we know the new variable cost will increase by a third, then the new variable cost will be $3 ($2.25 X
133%). Given that the contribution margin rate is 70%, variable cost will be 30% of the sales price, so $3
variable cost / 30% = $10 unit sales price.

D. See correct answer.

Question 105 - CMA 694 4-27 - Decision Making

Condensed monthly operating income data for Korbin Inc. for May follows:

Urban Store Suburban Store Total


Sales $80,000 $120,000 $200,000
Variable costs 32,000 84,000 116,000
Contribution margin 48,000 36,000 84,000
Direct fixed costs 20,000 40,000 60,000
Store segment margin 28,000 (4,000) 24,000
Common fixed cost 4,000 6,000 10,000
Operating income 24,000 (10,000) 14,000

Additional information regarding Korbin's operations follows:


One-fourth of each store's direct fixed costs would continue if either store were closed.
Korbin allocates common fixed costs to each store on the basis of sales dollars.
Management estimates that closing the Suburban Store would result in a 10% decrease in the Urban Store's
sales, while closing the Urban Store would not affect the Suburban Store's sales.
The operating results for May are representative of all months.

One-half of the Suburban Store's dollar sales are from items sold at variable cost to attract customers to the store.
Korbin is considering the deletion of these items, a move that would reduce the Suburban Store's direct fixed
expenses by 15% and result in a 20% loss of Suburban Store's remaining sales volume. This change would not affect
the Urban Store. A decision by Korbin to eliminate the items sold at cost would result in a monthly increase (decrease)
in Korbin's operating income of

A. $(7,200)
B. $2,000
C. $(5,200)
D. $(1,200)

A. This answer represents the reduction in CM ($36,000 X 20%).

(c) HOCK international, page 65


Part 3 : 11/11/10 07:42:55

B. The correct answer is $(1,200).

C. This answer $(5,200) represents the new segment margin ($28,800 - $34,000).

D. Since 50% of the sales are derived from selling at VC, the CM must be made up from the other sales.
Therefore, if the other sales decrease by 20%, then CM must decrease by $7,200 ($36,000 X 20%). If fixed cost
decrease by 15%, or $6,000 ($40,000 X 15%) then the monthly decrease in Korbin's operating income would be
$1,200.

Question 106 - CMA 687 5-21 - Decision Making

Gleason Co. has two products, a frozen dessert and ready-to-bake breakfast rolls, ready for introduction. However,
plant capacity is limited, and only one product can be introduced at present. Therefore, Gleason has conducted a
market study, at a cost of $26,000, to determine which product will be more profitable. The results of the study follow.

Sales of Desserts Sales of Rolls


at $1.80/unit at $1.20/unit
Volume Probability Volume Probability
250,000 .30 200,000 .20
300,000 .40 250,000 .50
350,000 .20 300,000 .20
400,000 .10 350,000 .10

The costs associated with the two products have been estimated by Gleason's cost accounting department and are
shown as follows.

Dessert Rolls
Ingredients per unit $ .40 $ .25
Direct labor per unit .35 .30
Variable overhead per unit .40 .20
Production tooling* 48,000 25,000
Advertising 30,000 20,000

*Gleason treats production tooling as a current operating expense rather than capitalizing it as a fixed asset.

According to Gleason's market study, the expected value of the sales volume of the breakfast rolls is

A. 125,000 units.
B. 260,000 units.
C. Some amount other than those given.
D. 275,000 units.

A. The answer of 125,000 was calculated by using the estimated sales volume of 250,000 times the probability of .5.
This is the incorrect calculation.

B. The calculation to find the expected sales volume is to multiply the estimated sells volume by the
probability [(200,000 X .2) + (250,000 X .5) + (300,000 X .2) + (350,000 X .1)]. Expected sales volume is 260,000
units.

C. See the correct answer.

D. This calculation represents the average sales volume of the Rolls.

(c) HOCK international, page 66


Part 3 : 11/11/10 07:42:55

Question 107 - CIA 1195 IV-75 - Decision Making

Which of the following price adjustment strategies is designed to stabilize production for the selling firm?

A. Seasonal discounts.
B. Cash discounts.
C. Quantity discounts.
D. Functional discounts.

A. Seasonal discounts are a good method by manufactures to smooth/stabilize the production cycle. The best
example is ski manufactures offering discounts to retailers during the spring and summer months. This will
encourage the retailers to order early.

B. A cash discount strategy is used in order to get customers to make timely payments.

C. Quantity discounts is used in order to get customers to make high volume purchases.

D. Functional discounts are given to customers for performing certain functions, such as selling, storing, etc. They are
not methods to smooth the production cycle.

Question 108 - CIA 1193 IV-24 - Decision Making

There is a market for both product X and product Y. Which of the following costs and revenues would be most relevant
in deciding whether to sell product X or process it further to make product Y?

A. Additional cost of making Y, given the cost of making X, and additional revenue from Y.
B. Additional cost of making X, given the cost of making Y, and additional revenue from Y.
C. Total cost of making X and the revenue from sale of X and Y.
D. Total cost of making Y and the revenue from sale of Y.

A. Incremental (differential) cost look only at relevant cost, cost that will differ among the related options. The
relevant cost in this problem are the additional cost of making product Y, and the additional revenue of selling
product Y.

B. Product Y can only be processed after product X is completed.

C. The cost of making product X is a sunk cost and not relevant.

D. Only incremental (differential) cost are considered relevant.

Question 109 - CMA 1290 4-10 - Decision Making

Costs relevant to a make-or-buy decision include variable manufacturing costs as well as

A. Avoidable fixed costs.


B. Property taxes.
C. Factory depreciation.
D. Unavoidable costs.

(c) HOCK international, page 67


Part 3 : 11/11/10 07:42:55

A. Relevant costs are those costs that differ among related choices. Thus, the only costs that are relevant in a
make-or-buy decision are avoidable variable avoidable fixed costs.

B. Property taxes are fixed costs, so therefore, not relevant to the decision.

C. Factory depreciation is a fixed cost and is not relevant, unless it is avoidable.

D. Unavoidable costs will continue regardless of the decision to make-or-buy, so therefore, these costs are not
relevant to the decision.

Question 110 - CMA Sample Q4-3 - Decision Making

Polar Company sells refrigeration components both in the U.S. and to a subsidiary located in France. One of the
components, Part No. 456, has a variable manufacturing cost of $30. The part can be sold domestically or shipped to
the French subsidiary for use in the manufacture of a residential subassembly. Relevant data with regard to Part No.
456 are shown below.

Part No. 456


Domestic selling price $65
Shipping charges to France 15
Cost of acquiring Part No. 456
in France 75
French residential subassembly
Sales price 170
Other additional manufacturing costs 55
Units shipped to France 150,000*

*If deemed preferable, these units could be sold in the U.S.

Polar's applicable income tax rates are 40% in the U.S. and 70% in France.

Polar will transfer Part No. 456 to the French subsidiary at either variable manufacturing cost or the domestic market
price. On the basis of this information, which one of the following strategies should be recommended to Polar's
management?

A. Sell 150,000 units in the U.S. and the French subsidiary obtains Part No. 456 in France.
B. Transfer 150,000 units at $65 and have the U.S. company absorb the shipping costs.
C. Transfer 150,000 units at $30 and the French subsidiary pays the shipping costs.
D. Transfer 150,000 units at $65 and the French subsidiary pays the shipping costs.

A.

The correct way to approach this problem is to calculate the per unit contribution margin for both companies
(U.S. and France) for each of the four alternatives, then subtract the applicable taxes for both companies for
the four alternatives and total the after-tax contribution margins for both companies for each of the four
alternatives. The highest total after-tax contribution margin is the best alternative.

For this alternative, the schedule is as follows:


Whole
US France Company

Selling price
$65.00 $170.00
Variable costs
30.00 75.00

(c) HOCK international, page 68


Part 3 : 11/11/10 07:42:55

Other manufacturing costs


-- 55.00
Shipping expense
-- --
---------- ------------
Unit contribution margin before tax $35.00 $40.00
Tax 14.00 28.00
---------- ------------ ------------
Net incremental profit per unit
$21.00 $12.00 $33.00

This is the highest net incremental profit of the four alternatives. As long as both companies (US and France)
have positive after-tax contributions, then the company is generally better off if they each buy and sell
separately.

B.

The correct way to approach this problem is to calculate the per unit contribution margin for both companies (U.S. and
France) for each of the four alternatives, then subtract the applicable taxes for both companies for the four alternatives
and total the after-tax contribution margins for both companies for each of the four alternatives. The highest total
after-tax contribution margin is the best alternative.

For this alternative, the schedule is as follows:


Whole
US France Company

Selling price
$65.00 $170.00
Variable costs
30.00 65.00
Other manufacturing costs
-- 55.00
Shipping expense
15.00 --
---------- ------------
Unit contribution margin before tax $20.00 $50.00
Tax 8.00 35.00
---------- ------------ ------------
Net incremental profit per unit
$12.00 $15.00 $27.00

This is not the highest possible incremental per unit profit for the company as a whole.

C.

The correct way to approach this problem is to calculate the per unit contribution margin for both companies (U.S. and
France) for each of the four alternatives, then subtract the applicable taxes for both companies for the four alternatives
and total the after-tax contribution margins for both companies for each of the four alternatives. The highest total
after-tax contribution margin is the best alternative.

For this alternative, the schedule is as follows:


Whole
US France Company

(c) HOCK international, page 69


Part 3 : 11/11/10 07:42:55

Selling price
$30.00 $170.00
Variable costs
30.00 30.00
Other manufacturing costs
-- 55.00
Shipping expense
-- 15.00
---------- ------------
Unit contribution margin before tax $0.00 $70.00
Tax 0.00 49.00
---------- ------------ ------------
Net incremental profit per unit
$0.00 $21.00 $21.00

This is not the highest possible incremental per unit profit for the company as a whole.

D.

The correct way to approach this problem is to calculate the per unit contribution margin for both companies (U.S. and
France) for each of the four alternatives, then subtract the applicable taxes for both companies for the four alternatives
and total the after-tax contribution margins for both companies for each of the four alternatives. The highest total
after-tax contribution margin is the best alternative.

For this alternative, the schedule is as follows:


Whole
US France Company

Selling price
$65.00 $170.00
Variable costs
30.00 65.00
Other manufacturing costs
-- 55.00
Shipping expense
-- 15.00
---------- ------------
Unit contribution margin before tax $35.00 $35.00
Tax 14.00 24.50
---------- ------------ ------------
Net incremental profit per unit
$21.00 $10.50 $31.50

This is not the highest possible incremental per unit profit for the company as a whole.

Question 111 - CMA 690 5-26 - Decision Making

Which one of the following costs would be relevant in short-term decision making?

A. Total variable costs that are the same in the considered alternatives.
B. Incremental fixed costs.
C. Opportunity costs that are the same in the considered alternatives.
D. All costs of inventory.

(c) HOCK international, page 70


Part 3 : 11/11/10 07:42:55

A. If variable cost are the same in the considered alternatives then these cost are not relevant.

B. Incremental (differential) costs are additional costs that result when production, or some other factor is
increased. Incremental cost can be either fixed or variable, so in this case incremental fixed cost would be
relevant in the short-term.

C. If opportunity cost are the same in the considered alternatives then they would not relevant.

D. It could be possible that inventory cost might not differ among considered alternatives.

Question 112 - CMA 687 5-20 - Decision Making

Cohasset Company currently manufactures all component parts used in the manufacture of various hand tools. A
handle is used in three different tools. The unit cost budget for 20,000 handles is

Direct material $ .60


Direct labor .40
Variable overhead .10
Fixed overhead .20
Total unit cost $1.30

R&M Steel has offered to supply 20,000 handles to Cohasset for $1.25 each, delivered. If Cohasset currently has idle
capacity that cannot be used, accepting the offer will

A. Increase the handle unit cost by $.05.


B. Increase the handle unit cost by $.15.
C. Decrease the handle unit cost by $.15.
D. Decrease the handle unit cost by $.05.

A. The company is better off as long as the company has idle capacity and the suppliers cost is above the company's
total VC.

B. As long as the company has idle capacity the company would be better off to produce the handles as long
as the suppliers price is above total VC ($1.10). If Cohasset's price is $1.25 then buying the handles would
increase the handle cost by $.15 per handle ($1.25 - $1.10).

C. The company is better off as long as the company has idle capacity and the suppliers cost is above the company's
total VC.

D. The company is better off as long as the company has idle capacity and the suppliers cost is above the company's
total VC.

Question 113 - CMA 1285 5-30 - Decision Making

The opportunity cost of making a component part in a factory with excess capacity for which there is no alternative use
is

A. The total manufacturing cost of the component.


B. The total variable cost of the component.
C. Zero.
D. The fixed manufacturing cost of the component.

(c) HOCK international, page 71


Part 3 : 11/11/10 07:42:55

A. Opportunity cost has to do with lost benefit (inflow). Manufacturing cost is an outflow.

B. Opportunity cost has to do with lost benefit (inflow). Total variable cost is an outflows.

C. The key term is "no alternative use." Opportunity cost is the foregone benefit by not selecting the next best
use of a resource. Therefore, opportunity cost is zero since there is no alternative use of plant capacity.

D. Opportunity cost has to do with lost benefit (inflow). Fixed manufacturing cost is an outflow.

Question 114 - CMA 1292 4-1 - Decision Making

The opportunity cost of making a component part in a factory with no excess capacity is the

A. Cost of the production given up in order to manufacture the component.


B. Fixed manufacturing cost of the component.
C. Net benefit given up from the best alternative use of the capacity.
D. Variable manufacturing cost of the component.

A. Opportunity cost is the lost contribution by not using a limited resource for its best alternative. It is not a cost.

B. Opportunity cost is the lost contribution by not using a limited resource for its best alternative. It is not fixed
manufacturing cost.

C. If there is an "alternative use" of the capacity then opportunity cost would be the lost contribution by not
selecting an "alternative use" of the capacity.

D. Opportunity cost is the lost contribution by not using a limited resource for its best alternative. It is not variable
manufacturing cost.

Question 115 - CIA 593 IV-17 - Decision Making

A company with $280,000 of fixed costs has the following data:

Product A Product B
Sales price per unit $5 $6
Variable costs per unit $3 $5

Assume three units of A are sold for each unit of B sold. How much will sales be in dollars of product B at the
breakeven point?

A. $200,000
B. $280,000
C. $840,000
D. $240,000

A. See correct answer.

B. See correct answer.

C. See correct answer.

(c) HOCK international, page 72


Part 3 : 11/11/10 07:42:55

D. First step is to calculate the composite units contribution margin for products A & B. The formula is: = [3
units of A x ($5 - $3)] + [1 unit of B x ($6 - $5)]. =(3 X $2) + (1 X $1) = $6 + $1 = $7 composite unit contribution
margin. Second step is to calculate the composite units. Fixed cost$280,000 =Composite CM$7 = 40,000
composite units Third step is to calculate the breakeven point in sales. = 40,000 units x $6 sales price of
product B. = $240,000

Question 116 - CMA 687 5-22 - Decision Making

Gleason Co. has two products, a frozen dessert and ready-to-bake breakfast rolls, ready for introduction. However,
plant capacity is limited, and only one product can be introduced at present. Therefore, Gleason has conducted a
market study, at a cost of $26,000, to determine which product will be more profitable. The results of the study follow.

Sales of Desserts Sales of Rolls


at $1.80/unit at $1.20/unit
Volume Probability Volume Probability
250,000 .30 200,000 .20
300,000 .40 250,000 .50
350,000 .20 300,000 .20
400,000 .10 350,000 .10

The costs associated with the two products have been estimated by Gleason's cost accounting department and are
shown as follows.

Dessert Rolls
Ingredients per unit $ .40 $ .25
Direct labor per unit .35 .30
Variable overhead per unit .40 .20
Production tooling* 48,000 25,000
Advertising 30,000 20,000

*Gleason treats production tooling as a current operating expense rather than capitalizing it as a fixed asset.

Applying a deterministic approach, Gleason's revenue from sales of frozen desserts would be

A. Some amount other than those given.


B. $540,000.
C. $549,000.
D. $216,000.

A. See the correct answer.

B. The deterministic approach uses the most probable value to calculate revenue. In this case volume of
300,000 is the most probable, so expected revenue would be $540,000 (300,000 units X $1.80/unit).

C. This would be the answer if using the probabilistic approach. Though, the question applied to the deterministic
approach.

D. The answer incorrectly used the probability of having a sells value of 300,000 units (300,000 X .4 X $1.80/unit).

Question 117 - CMA 694 4-30 - Decision Making

(c) HOCK international, page 73


Part 3 : 11/11/10 07:42:55

Bruell Electronics Co. is developing a new product, surge protectors for high-voltage electrical flows. The following
cost information relates to the product:

Unit Costs
Direct materials $3.25
Direct labor 4.00
Distribution .75

The company will also be absorbing $120,000 of additional fixed costs associated with this new product. A corporate
fixed charge of $20,000 currently absorbed by other products will be allocated to this new product.

How many surge protectors (rounded to the nearest hundred) must Bruell Electronics sell at a selling price of $14 per
unit to increase after-tax income by $30,000? Bruell Electronics' effective income tax rate is 40%.

A. 12,100 units.
B. 28,300 units.
C. 20,000 units.
D. 10,700 units.

A. See the correct answer.

B. The breakeven point is calculated by dividing the additional fixed costs plus the desired pre-tax income by
the contribution margin. It is important to note that the given desired after-tax income is $30,000. To calculate
the answer, you need to find the pre-tax income. The pre-tax income is $50,000 ($30,000 / (1 - .40). Fixed costs
+ the desired pre-tax income contribution margin = $120,000 + $50,000, or $170,000. $170,000 / [$14 - ($3.25 +
$4.00 + $.75)] = 28,333 units, rounded to $28,300 units.

C. See the correct answer.

D. See the correct answer.

Question 118 - CMA 1296 4-3 - Decision Making

Kator Co. is a manufacturer of industrial components. One of their products that is used as a subcomponent in auto
manufacturing is KB-96. This product has the following financial structure per unit:

Selling price $150


Direct materials $20
Direct labor 15
Variable manufacturing overhead 12
Fixed manufacturing overhead 30
Shipping and handling 3
Fixed selling and administrative 10
Total costs $90

During the next year, KB-96 sales are expected to be 10,000 units. All of the costs will remain the same except that
fixed manufacturing overhead will increase by 20% and direct materials will increase by 10%. The selling price per unit
for next year will be $160. Based on this data, the contribution margin from KB-96 for next year will be

A. $1,110,000
B. $750,000
C. $1,080,000
D. $620,000

(c) HOCK international, page 74


Part 3 : 11/11/10 07:42:55

A. See correct answer.

B. See correct answer.

C. Contribution margin formula is sales price per unit - all variable cost. Sales price =$160 Less: Variable Cost
Direct Material ($20 X 10% increase in cost) =$22 Direct Labor =$15 Variable overhead =$12 Shipping and
Handling =$3 Total Variable cost =$52 Contribution margin (per unit) =$108 Number of units sold during year
=10,0000 Total Contribution margin =$1,080,000

D. See correct answer.

Question 119 - CIA 591 IV-17 - Decision Making

A company makes a product that sells for $30. During the coming year, fixed costs are expected to be $180,000, and
variable costs are estimated at $26 per unit. How many units must the company sell to break even?

A. 6,000
B. 720,000
C. 45,000
D. 6,924

A. 6,000 units of sales would only be enough to cover fixed costs, not its variable costs ($180,000 Fixed Cost / $30
sales price).

B. See correct answer.

C. Breakeven point is found by dividing fixed costs by the contribution margin. $180,000 Fixed Costs $4
Contribution Margin =45,000 units

D. This answer was found by dividing fixed costs by its variable costs. Need to divide by the contribution margin.

Question 120 - CMA 1290 4-12 - Decision Making

Opportunity costs are

A. Relevant to decision making.


B. Not used for decision making.
C. Equal to historical costs.
D. The same as variable costs.

A. Opportunity cost is relevant to decision making. Opportunity cost is the benefit to income (inflow) that is
lost by not using a limited resource for its best alternative use.

B. Opportunity cost are relevant to the decision making process.

C. Historical cost are those cost that have been incurred in the past.

D. Opportunity cost is the benefit to income (inflow) that is lost by not using a limited resource for its best alternative
use. Variable cost relate only to those cost that vary with the level of production (outflow).

(c) HOCK international, page 75


Part 3 : 11/11/10 07:42:55

Question 121 - CIA 585 IV-8 - Decision Making

Orange Company's controller developed the following direct-costing income statement for year 1:

Per Unit
Sales (150,000 units at $30) $4,500,000 $30
Variable costs:
Direct materials $1,050,000 $7
Direct labor 1,500,000 10
Mfg. overhead 300,000 2
Selling & mkg. 300,000 2
(3,150,000) $21
Contribution margin $1,350,000 $9
Fixed costs:
Mfg. overhead $600,000 $4
Selling & mkg. 300,000 2
(900,000) $6
Net income $450,000 $3

Orange Co. based its next year's budget on the assumption that fixed costs, unit sales, and the sales price would
remain as they were in year 1, but with net income being reduced to $300,000. By July of year 2, the controller was
able to predict that unit sales would increase over year 1 levels by 10%. Based on the year 2 budget and the new
information, the predicted year 2 net income would be

A. $330,000
B. $420,000
C. $300,000
D. $585,000

A. This answer is incorrect since it is also including a 10% increase in fixed costs as well.

B. Based on the information provided, if profit is reduced to $300,000, then the contribution margin in year 1
would be $1,200,000 ($300,000 profit + $900,000). If sales level is expected to increase by 10% then the
contribution margin in year 2 would be expected to be $1,320,000 ($1,200,000 X 110%). The new expected
profit in year 2 would be $420,000 ($1,320,000 - $900,000 fixed costs).

C. This is the profit in year 1. It is predicted that the sales level would be increased by 10% in year 2.

D. This unit contribution margin in year 2 will be $8, not $9.

Question 122 - CMA 691 4-11 - Decision Making

If inventories are expected to change, the type of costing that provides the best information for breakeven analysis is

A. Absorption (full) costing.


B. Joint costing.
C. Variable (direct) costing.
D. Job order costing.

A. Absorption costing includes both fixed and variable costs. They are not separated.

B. Variable cost in joint costing are not separated from fixed.

(c) HOCK international, page 76


Part 3 : 11/11/10 07:42:55

C. Variable costing is the best method of costing. In the CVP formula, fixed cost is separated from variable
cost.

D. Variable cost in job order costing are not separated from fixed cost.

Question 123 - CMA 690 5-14 - Decision Making

Madengrad Company manufactures a single electronic product called Precisionmix. This unit is a batch-density
monitoring device attached to large industrial mixing machines used in flour, rubber, petroleum, and chemical
manufacturing. Precisionmix sells for $900 per unit. The following variable costs are incurred to produce each
Precisionmix device:

Direct labor $180


Direct materials 240
Factory overhead 105
Total variable production costs 525
Marketing costs 75
Total variable costs $600

Madengrad's income tax rate is 40%, and annual fixed costs are $6,600,000. Except for an operating loss incurred in
the year of incorporation, the firm has been profitable over the last 5 years.

For Madengrad Company to achieve an after-tax net income of $540,000, annual sales revenue must be

A. $23,850,000
B. $2,700,000
C. $21,420,000
D. $22,500,000

A. See correct answer.

B. See correct answer.

C. See correct answer.

D. = $22,500,000

Question 124 - CMA 692 4-22 - Decision Making

Mason Enterprises has prepared the following budget for the month of July.

Selling Price Variable Cost Sales


Per Unit Per Unit Unit
Product A $10.00 $4.00 15,000
Product B 15.00 8.00 20,000
Product C 18.00 9.00 5,000

Assuming that total fixed costs will be $150,000 and the mix remains constant, the breakeven point (rounded to the
next higher whole unit) will be

A. 21,429 units.

(c) HOCK international, page 77


Part 3 : 11/11/10 07:42:55

B. 21,819 units.
C. 6,818 units.
D. 20,455 units.

A. See correct answer.

B. The sales ratio will be 3:4:1 (15,000 units (Product A), 20,000 units (Product B, 5,000 units (Product C)).
Given the new ratio the breakeven volume is: 21,819 units. Product A Product B Product C Budgeted sales
ratio 3 4 1 Sales price $10.00 $15.00 $18.00 Variable costs $4.00 $8.00 $9.00 Contribution margin $6.00 $7.00
$9.00 Composite UCM 18 (6 x 3) 28 (7 x 4) 9 (9 x 1) Composite UCM =55 (18 + 28 + 9) Fixed costs Composite
UCM =$150,000/ 55 =2,728 composite units X 8 units (3 + 4 + 1) =21,819 total units to breakeven (rounded up)

C. See correct answer.

D. See correct answer.

Question 125 - CIA 1185 IV-9 - Decision Making

A manufacturer produces a product that sells for $10 per unit. Variable costs per unit are $6 and total fixed costs are
$12,000. At this selling price, the company earns a profit equal to 10% of total dollar sales. By reducing its selling price
to $9 per unit, the manufacturer can increase its unit sales volume by 25%. Assume that there are no taxes and that
total fixed costs and variable costs per unit remain unchanged. If the selling price were reduced to $9 per unit, the
profit would be

A. $3,000
B. $6,000
C. $5,000
D. $4,000

A. The answer to find the new expected profit is found first by calculating the past periods sales volume
(given a 10% profit of sales revenue). Sales $10 Less: VC $6 CM $4 = $12,000 fixed costs $4 CM - ($10(10%
profit)) = $12,000 fixed costs $3 = 4,000 units sold last Sales volume is expected to increase by 25%, so the
new sales volume will be 5,000 units (4,000 X 125%). Sales $9 Less: VC $6 CM $3 Dollar CM $15,000 (5,000 x
$3 CM) Less: Fixed costs $12,000 Profit $3,000

B. See correct answer.

C. See correct answer.

D. See correct answer.

Question 126 - CIA 577 IV-11 - Decision Making

Which of the following will result in raising the breakeven point?

A. A decrease in the variable cost per unit.


B. An increase in the semivariable cost per unit.
C. A decrease in income tax rates.
D. An increase in the contribution margin per unit.

A. A decrease in variable cost per unit would increase the contribution margin, thereby, decrease the breakeven point.

(c) HOCK international, page 78


Part 3 : 11/11/10 07:42:55

B. Semivariable cost per unit can include either fixed, or variable and an increase in either will raise the
breakeven point.

C. Income taxes are considered to be a variable expenses, and as such a decrease in the income tax rate would
increase the contribution margin, thereby, lowering the breakeven point.

D. An increase in the contribution margin per unit would decrease the breakeven point.

Question 127 - CMA 692 4-25 - Decision Making

Laurel Corporation has its own cafeteria with the following annual costs:

Food $100,000
Labor 75,000
Overhead 110,000
Total $285,000

The overhead is 40% fixed. Of the fixed overhead, $25,000 is the salary of the cafeteria supervisor. The remainder of
the fixed overhead has been allocated from total company overhead. Assuming the cafeteria supervisor will remain
and the Laurel will continue to pay his/her salary, the maximum cost Laurel will be willing to pay an outside firm to
service the cafeteria is

A. $219,000.
B. $285,000.
C. $175,000.
D. $241,000.

A. This answer incorrectly assumes that fixed overhead cost are avoidable $44,000 ($110,000 X 40%). Fixed
overhead cost are unavoidable.

B. What to look at avoidable cost. The $285,000 is total cost of which $219,000 will be avoidable if getting service
from the outside.

C. This is incorrect since the $175,000 includes only food and labor. It does not include avoidable overhead cost.

D. In order to find what is the most Laurel would be willing to pay an outside company need to look at
avoidable variable and fixed cost. Total avoidable cost is $241,000 ($100,000 food + $75,000 labor + 66,000 of
avoidable overhead).

Question 128 - CMA 690 4-27 - Decision Making

When a fixed plant asset with a 5-year estimated useful life is sold during the second year, how would the use of an
accelerated depreciation method instead of the straight-line method affect the gain or loss on the sale of the fixed
plant asset?

GainLoss

A. DecreaseDecrease
B. IncreaseDecrease
C. DecreaseIncrease
D. IncreaseIncrease

(c) HOCK international, page 79


Part 3 : 11/11/10 07:42:55

A. The accelerated method would cause the gain to be greater, not less.

B. The accelerated method will depreciate the asset quicker in the early years, therefore increasing the gain.
On the other hand the loss would be less since the book value would be less then under the straight-line
method.

C. The opposite would happen.

D. The use of an accelerated method would decrease the loss since assets are depreciated at a faster rate in the
early years.

Question 129 - CMA 687 5-26 - Decision Making

Gleason Co. has two products, a frozen dessert and ready-to-bake breakfast rolls, ready for introduction. However,
plant capacity is limited, and only one product can be introduced at present. Therefore, Gleason has conducted a
market study, at a cost of $26,000, to determine which product will be more profitable. The results of the study follow.

Sales of Desserts Sales of Rolls


at $1.80/unit at $1.20/unit
Volume Probability Volume Probability
250,000 .30 200,000 .20
300,000 .40 250,000 .50
350,000 .20 300,000 .20
400,000 .10 350,000 .10

The costs associated with the two products have been estimated by Gleason's cost accounting department and are
shown as follows.

Dessert Rolls
Ingredients per unit $ .40 $ .25
Direct labor per unit .35 .30
Variable overhead per unit .40 .20
Production tooling* 48,000 25,000
Advertising 30,000 20,000

*Gleason treats production tooling as a current operating expense rather than capitalizing it as a fixed asset.

The advertising expense estimated by Gleason for the introduction of the new products is an example of a(n)

A. Opportunity cost.
B. Committed cost.
C. Discretionary cost.
D. Conversion cost.

A. Opportunity cost is the lost revenue of not selecting the next best alternative. This does not relate to the cost of
advertising.

B. Committed cost are those cost that have not yet been spent, but have been committed to the company. Advertising
is a discretionary cost.

C. Discretionary cost are fixed cost that are necessary to operate, but do not have a clear relationship
between input and output. Advertising in this problem would be an example of a discretionary cost.

D. Conversion cost include labor and overhead cost. Advertising is an example of a discretionary cost.

(c) HOCK international, page 80


Part 3 : 11/11/10 07:42:55

Question 130 - CMA 696 4-18 - Decision Making

Which one of the following is most relevant to a manufacturing equipment replacement decision?

A. Gain or loss on the disposal of the old equipment.


B. Original cost of the old equipment.
C. Disposal price of the old equipment.
D. A lump-sum write-off amount from the disposal of the old equipment.

A. Gain or loss on sale of asset is based on historical cost information, which is a sunk cost and not relevant in the
decision.

B. Original cost of the equipment is based on historical cost, which is a sunk cost and therefore not relevant to the
decision.

C. The cash inflow that the company would be able to gain from disposing of the old piece of equipment is
most relevant because this cash inflow will only occur if the decision is to dispose of the old piece of
equipment.

D. A lump-sum write-off of an asset is a sunk cost and not relevant to the decision.

Question 131 - CMA 1273 4-4 - Decision Making

Which of the following would decrease unit contribution margin the most?

A. A 15% decrease in selling price.


B. A 15% increase in variable expenses.
C. A 15% decrease in variable expenses.
D. A 15% decrease in fixed expenses.

A. Assuming that the selling price is greater than variable expenses, a 15% decrease in selling price would
have the greatest impact on contribution margin.

B. A 15% increase in variable expenses would not have as great an impact on the contribution margin as a 15%
decrease in selling price.

C. Decreasing variable expenses will increase the contribution margin.

D. Changing fixed expenses will have no effect on the contribution margin.

Question 132 - CMA 694 4-28 - Decision Making

Bruell Electronics Co. is developing a new product, surge protectors for high-voltage electrical flows. The following
cost information relates to the product:

Unit Costs
Direct materials $3.25
Direct labor 4.00

(c) HOCK international, page 81


Part 3 : 11/11/10 07:42:55

Distribution .75

The company will also be absorbing $120,000 of additional fixed costs associated with this new product. A corporate
fixed charge of $20,000 currently absorbed by other products will be allocated to this new product.

If the selling price is $14 per unit, the breakeven point in units (rounded to the nearest hundred) for surge protectors is

A. 20,000 units.
B. 15,000 units.
C. 10,000 units.
D. 8,500 units.

A. The breakeven point is calculated by dividing the additional fixed costs by the contribution margin. Fixed
costs contribution margin =$120,000 [$14 - ($3.25 + $4.00 + $.75)] = 20,000 units

B. See correct answer.

C. See correct answer.

D. See correct answer.

Question 133 - CMA 691 4-15 - Decision Making

Regis Company manufactures plugs used in its manufacturing cycle at a cost of $36 per unit that includes $8 of fixed
overhead. Regis needs 30,000 of these plugs annually, and Orlan Company has offered to sell these units to Regis at
$33 per unit. If Regis decides to purchase the plugs, $60,000 of the annual fixed overhead applied will be eliminated,
and the company may be able to rent the facility previously used for manufacturing the plugs.

If the plugs are purchased and the facility rented, Regis Company wishes to realize $100,000 in savings annually. To
achieve this goal, the minimum annual rent on the facility must be

A. $190,000.
B. $10,000.
C. $70,000.
D. $40,000.

A. If purchasing the plugs, Regis will have a total loss of $90,000 (30,000 units X $3.00 loss if purchasing the
plugs) if purchasing from an outside supplier. If Regis wishes to realize a profit of $100,000 then the minimum
rent fee must be $190,000 (-$90,000 + X = $100,000).

B. See correct answer.

C. See correct answer.

D. See correct answer.

Question 134 - CMA 1296 4-6 - Decision Making

Several surveys point out that most managers use full product costs, including unit fixed costs and unit variable costs,
in developing cost-based pricing. Which one of the following is least associated with cost-based pricing?

A. Price stability.

(c) HOCK international, page 82


Part 3 : 11/11/10 07:42:55

B. Price justification.
C. Fixed-cost recovery.
D. Target pricing.

A. Full product costing promotes price stability.

B. Price justification is used in antitrust cases where the seller has to justify the price it is charging to customers.

C. Under fixed-cost recovery all costs in the long-term are relevant and have to be recovered.

D. Target pricing starts with the expected price the product should be sold in the market, based on its
knowledge of the customers and competitors. By subtracting the per unit profit margin the seller is able to
estimate the unit target cost. If the seller does not believe that it would be able to produce the product at, or
below the target cost the project would abandoned.

Question 135 - CMA 1296 4-4 - Decision Making

Kator Co. is a manufacturer of industrial components. One of their products that is used as a subcomponent in auto
manufacturing is KB-96. This product has the following financial structure per unit:

Selling price $150


Direct materials $20
Direct labor 15
Variable manufacturing overhead 12
Fixed manufacturing overhead 30
Shipping and handling 3
Fixed selling and administrative 10
Total costs $90

Kator Co. has received a special, one-time, order for 1,000 KB-96 parts. Assume that Kator is operating at full
capacity and that the contribution margin of the output that would be displaced by the special order is $10,000. The
minimum price that is acceptable, using the original data, for this one-time special order is in excess of

A. $60
B. $70
C. $100
D. $87

A. If the company is operating at full capacity then in order to produce the special order the company will lose
$10,000 of potential contribution, or $10 per unit ($10,000 lost contribution / 1,000 units). This lost opportunity
has to be reflected in the price that will be charged for the special order, or $60 per unit ($20 DM + $15 DL +
$12 VOH + $3 shipping and handling + $10 opportunity cost).

B. This answer is including fixed selling and administrative cost which are unavoidable and should not be included.

C. The answer $100 is full costing the price ($90 full cost of KB-96 + $10 opportunity cost). The correct answer of $60
should include only incremental cost, plus the opportunity cost ($50 + $10)

D. The answer of $87 includes fixed manufacturing, but does not include shipping and handling.

Question 136 - CMA 1296 4-8 - Decision Making

(c) HOCK international, page 83


Part 3 : 11/11/10 07:42:55

Whitehall Corporation produces chemicals used in the cleaning industry. During the previous month, Whitehall
incurred $300,000 of joint costs in producing 60,000 units of AM-12 and 40,000 units of BM-36. Whitehall uses the
units-of-production method to allocate joint costs. Currently, AM-12 is sold at split-off for $3.50 per unit. Flank
Corporation has approached Whitehall to purchase all of the production of AM-12 after further processing. The further
processing will cost Whitehall $90,000.

Assume that Whitehall Corporation agreed to sell AM-12 to Flank Corporation for $5.50 per unit after further
processing. During the first month of production, Whitehall sold 50,000 units with 10,000 units remaining in inventory
at the end of the month. With respect to AM-12, which one of the following statements is correct?

A. The operating profit last month was $50,000, and the inventory value is $45,000.
B. The operating profit last month was $50,000, and the inventory value is $15,000.
C. The operating profit last month was $125,000, and the inventory value is $30,000.
D. The operating profit last month was $200,000, and the inventory value is $30,000.

A. If processing further the production cost of AM-12 is $4.50 per unit ($3.00 joint cost at split-off [$300,000 /
(60,000 units of AM-12 + 40,000 units of BM-36)] + $1.50 additional cost). Total profit margin per unit would be
$1.00 per unit ($5.50 sell price - $4.50 product cost). The gross profit would be $50,000 (50,000 units X $1.00
per unit profit). If 10,000 units are in inventory and the per unit cost of production is $4.50 than the value of
product in inventory must be $45,000 ($4.50 X 10,000).

B. See correct answer.

C. See correct answer.

D. See correct answer.

Question 137 - CMA 1295 4-21 - Decision Making

A decision-making concept, described as "the contribution to income that is forgone by not using a limited resource for
its best alternative use," is called

A. Opportunity cost.
B. Incremental cost.
C. Marginal cost.
D. Potential cost.

A. This is the definition of Opportunity cost.

B. Incremental cost is the cost difference between two related options.

C. Marginal cost is the cost of producing one more unit.

D. Potential cost is the cost that may be incurred at a future date.

Question 138 - CMA 696 4-19 - Decision Making

When an organization decides on a course of action that is selected from a group of alternative courses of action, the
benefit lost by not choosing the best alternative course of action is the

A. Net realizable value.

(c) HOCK international, page 84


Part 3 : 11/11/10 07:42:55

B. Expected value.
C. Opportunity cost.
D. Incremental cost.

A. Net realizable value is the value of an asset less any disposal cost.

B. The expected value of a product is the weighted average of the potential outcomes. It is calculated by taking the
expected volume of sales and multiplying by the estimated probabilities for each potential outcome and adding
together.

C. By definition, opportunity cost is the benefit to income (inflow) that is lost by not using a limited resource
for its best alternative use.

D. Incremental (differential) costs are additional costs that are incurred when production, or some other factor is
increased.

Question 139 - CMA 1290 4-3 - Decision Making

Relevant or differential cost analysis

A. Considers all variable and fixed costs as they change with each decision alternative.
B. Considers only variable costs as they change with each decision alternative.
C. Takes all variable and fixed costs into account to analyze decision alternatives.
D. Allows the decision maker to group all types of costs together to facilitate decision making.

A. Relevant and differential cost analysis looks at those cost that vary with the selected options.

B. It is possible that some fixed cost would be considered if they differ with the options.

C. Not all cost would be considered in relevant or differential cost analysis.

D. Those cost that would not vary among the selected options would be ignored.

Question 140 - CMA 696 4-15 - Decision Making

In a decision analysis situation, which one of the following costs is not likely to contain a variable cost component?

A. Depreciation.
B. Labor.
C. Selling.
D. Overhead.

A. Depreciation is a fixed cost that will not depend on the production level.

B. Manufacturing labor will depend on the production level, so is variable. Labor can also contain a fixed component,
such as management salaries.

C. Selling can be variable or mixed (variable and fixed).

D. Overhead can be variable or mixed (variable and fixed).

(c) HOCK international, page 85


Part 3 : 11/11/10 07:42:55

Question 141 - CMA 692 4-26 - Decision Making

J J Motors, Inc. employs 45 sales personnel to market its line of luxury automobiles. The average car sells for
$23,000, and a 6% commission is paid to the salesperson. J J Motors is considering a change to a commission
arrangement that would pay each salesperson a salary of $2,000 per month plus a commission of 2% of the sales
made by that salesperson. The amount of total monthly car sales at which J J Motors would be indifferent as to which
plan to select is

A. $1,250,000.
B. $3,000.000.
C. $2,250,000.
D. $1,500,000.

A. See the correct answer.

B. See the correct answer.

C.

There is more than one correct way to calculate the answer to this problem. One solution is to calculate the
sales volume where J J Motors will indifferent as to which plan is selected. Remember that the total sales
volume will be the sales made per salesperson multiplied by the number of sales personnel.

The solution can be calculated algebraically as follows:

Let X be the monthly sales per salesperson. Create an equation containing both possible payment methods
and set the two payment methods equal to one another. Solve for X, and then multiply the value you get for X
by 45 to get the total sales for all salespeople per month.

.06X = $2,000 + .02X

Simplify by subtracting .02X from both sides of the equation:

.04X = $2,000

Solve for X by dividing both sides of the equation by .04:

X = $50,000, which is the sales per salesperson at which the payment under both plans will be equal.

Multiply $50,000 by 45 salespeople to get the total sales per month at which both payment plans will result in
the same payments to salespeople:

$50,000 * 45 = $2,250,000

To check:

At a sales volume of $2,250,000, a straight commission of 6% will result in a total payment of $2,250,000 * .06,
or $135,000.

At a sales volume of $2,250,000, a salary of $2,000 per month paid to each salesperson plus a commission of
2% will result in a total payment of:

($2,000 * 45) + (.02 * $2,250,000) = $135,000

If you calculated the answer a different way but got the same answer, then your method is also correct.

(c) HOCK international, page 86


Part 3 : 11/11/10 07:42:55

D. See the correct answer.

Question 142 - CMA 687 5-24 - Decision Making

Gleason Co. has two products, a frozen dessert and ready-to-bake breakfast rolls, ready for introduction. However,
plant capacity is limited, and only one product can be introduced at present. Therefore, Gleason has conducted a
market study, at a cost of $26,000, to determine which product will be more profitable. The results of the study follow.

Sales of Desserts Sales of Rolls


at $1.80/unit at $1.20/unit
Volume Probability Volume Probability
250,000 .30 200,000 .20
300,000 .40 250,000 .50
350,000 .20 300,000 .20
400,000 .10 350,000 .10

The costs associated with the two products have been estimated by Gleason's cost accounting department and are
shown as follows.

Dessert Rolls
Ingredients per unit $ .40 $ .25
Direct labor per unit .35 .30
Variable overhead per unit .40 .20
Production tooling* 48,000 25,000
Advertising 30,000 20,000

*Gleason treats production tooling as a current operating expense rather than capitalizing it as a fixed asset.

In order to recover the costs of production tooling and advertising for the breakfast rolls, Gleason's sales of the
breakfast rolls would have to be

A. 37,500 units.
B. 60,000 units.
C. Some amount other than those given.
D. 100,000 units.

A. This answer is incorrect because it was calculated by dividing the $45,000 by the sale price of $1.20.

B. This answer is incorrect because it was calculated by dividing advertising and production tooling cost by variable
cost per unit [$45,000 - ($.25 - $.30 - $.20)].

C. See correct answer.

D. Total cost that will have to be recovered is $45,000 ($25,000 + $20,000). The contribution margin per unit is
$.45 ($1.20 - $.75). Dividing $45,000 / $.45 will give 100,000 units that will need sold to breakeven.

Question 143 - CMA 694 4-29 - Decision Making

Bruell Electronics Co. is developing a new product, surge protectors for high-voltage electrical flows. The following

(c) HOCK international, page 87


Part 3 : 11/11/10 07:42:55

cost information relates to the product:

Unit Costs
Direct materials $3.25
Direct labor 4.00
Distribution .75

The company will also be absorbing $120,000 of additional fixed costs associated with this new product. A corporate
fixed charge of $20,000 currently absorbed by other products will be allocated to this new product.

How many surge protectors (rounded to the nearest hundred) must Bruell Electronics sell at a selling price of $14 per
unit to gain $30,000 additional income before taxes?

A. 25,000 units.
B. 20,000 units.
C. 10,700 units.
D. 12,100 units.

A. The breakeven point is calculated by dividing the additional fixed costs plus the desired pre-tax income by
the contribution margin. Fixed costs + desired pre-tax income contribution margin =$120,000 + $30,000 [$14 -
($3.25 + $4.00 + $.75)] = 25,000 units

B. See correct answer.

C. See correct answer.

D. See correct answer.

Question 144 - CMA 1286 5-12 - Decision Making

Cost-volume-profit (CVP) analysis is a key factor in many decisions, including choice of product lines, pricing of
products, marketing strategy, and use of productive facilities. A calculation used in a CVP analysis is the breakeven
point. Once the breakeven point has been reached, operating income will increase by the

A. Fixed costs per unit for each additional unit sold.


B. Contribution margin per unit for each additional unit sold.
C. Gross margin per unit for each additional unit sold.
D. Variable costs per unit for each additional unit sold.

A. At breakeven, fixed costs have already been covered.

B. At the breakeven point, total fixed cost are being covered. Any unit sale above and beyond the breakeven
point will contribute to profit by the amount of each additional unit sold times the unit contribution margin.

C. Gross margin will include some fixed costs.

D. Operating income will increase by the amount of unit contribution margin times the number of units sold above the
breakeven point.

Question 145 - CIA 582 IV-23 - Decision Making

A company's breakeven point in sales dollars may be affected by equal percentage increases in both selling price and

(c) HOCK international, page 88


Part 3 : 11/11/10 07:42:55

variable cost per unit (assume all other factors are constant within the relevant range). The equal percentage changes
in selling price and variable cost per unit will cause the breakeven point in sales dollars to

A. Decrease by more than the percentage increase in the selling price.


B. Decrease by less than the percentage increase in selling price.
C. Increase by the percentage change in variable cost per unit.
D. Remain unchanged.

A. The breakeven point in sales revenue will remain unchanged.

B. The breakeven point in sales revenue will remain unchanged.

C. The breakeven point in sales revenue will remain unchanged.

D. The breakeven point in sales revenue is equal to fixed cost divided by the contribution margin percentage.
If the percentage per unit increase in variable and sales price are the same then the contribution margin
percentage will remain unchanged.

Question 146 - CMA 696 4-17 - Decision Making

When management must decide to accept or reject a one-time-only special order, given sufficient idle capacity, which
one of the following is not relevant to the decision?

A. Differential costs.
B. Absorption costs.
C. Direct costs.
D. Variable costs.

A. Differential (incremental) costs will differ among the decision choices, so therefore, relevant.

B. Relevant cost are those cost that will differ among the decision choices. Absorption costing method
includes some cost (fixed factory overhead) that will not change if the order is accepted or not.

C. Direct cost are cost that will be incurred only if the special order is accepted or rejected, so therefore, relevant.

D. Variable cost will be incurred only if the special order is accepted or rejected, so therefore, relevant.

Question 147 - CMA 690 5-24 - Decision Making

Relevant costs refer to

A. Costs that would be incurred within the relevant range of production.


B. Anticipated future costs that will differ among various alternatives.
C. Past costs that are expected to be different in the future.
D. All fixed costs.

A. Some cost may be shared cost so, therefore, would not be relevant, even within the relevant range of production.

B. Relevant cost are those types of costs that would differ among various courses of actions. For example, if
there are two options available to a manager, the manager would consider in his or her decision only those
cost that differ among the two options. Shared costs would not be considered since they are not relevant to
the decision.

(c) HOCK international, page 89


Part 3 : 11/11/10 07:42:55

C. Past cost are sunk cost and not relevant to the decision.

D. Relevant cost only refer to those cost that vary among the options. Some fixed cost may vary among the
alternatives and would be relevant, but not all fixed cost are relevant.

Question 148 - CIA 596 III-85 - Decision Making

A company that sells its single product for $40 per unit uses cost-volume-profit analysis in its planning. The company's
after-tax net income for the past year was $1,188,000 after applying an effective tax rate of 40%. The projected costs
for manufacturing and selling its single product in the coming year are shown below.

Variable cost per unit:


Direct material $5.00
Direct labor 4.00
Manufacturing overhead 6.00
Selling and administrative costs 3.00
Total variable cost per unit $18.00
Annual fixed operating costs:
Manufacturing overhead $6,200,000
Selling and administrative costs 3,700,000
Total annual fixed cost $9,900,000

The company has learned that a new direct material is available that will increase the quality of its product. The new
material will increase the direct material costs by $3 per unit. The company will increase the selling price of the
product to $50 per unit and increase its marketing costs by $1,575,000 to advertise the higher-quality product. The
number of units the company has to sell in order to earn a 10% before-tax return on sales would be

A. 337,500 units.
B. 346,875 units.
C. 478,125 units.
D. 425,000 units.

A. See correct answer.

B. See correct answer.

C. An increase in direct material costs of $3 will increase per unit variable costs from $18 per unit to $21 per
unit ($18 + $3). If the company is able to increase per unit sales price to $50 per unit then the per unit CM will
be $29. Sales -$50 Less: VC -$21 CM -$29 $9,900,000 + $1,575,000 (increase in marketing costs) =$24 [$29 CM -
$5 ($50 sales price X 10% return on sales)] =478,125 units

D. See correct answer.

Question 149 - CMA 693 4-1 - Decision Making

Delphi Company has developed a new project that will be marketed for the first time during the next fiscal year.
Although the Marketing Department estimates that 35,000 units could be sold at $36 per unit, Delphi's management
has allocated only enough manufacturing capacity to produce a maximum of 25,000 units of the new product annually.
The fixed costs associated with the new product are budgeted at $450,000 for the year, which includes $60,000 for
depreciation on new manufacturing equipment. Data associated with each unit of product are presented as follows.

(c) HOCK international, page 90


Part 3 : 11/11/10 07:42:55

Delphi is subject to a 40% income tax rate.

Variable Costs
Direct material $7.00
Direct labor 3.50
Manufacturing overhead 4.00
Total variable manufacturing cost 14.50
Selling expenses 1.50
Total variable cost $16.00

The number of units of the new product that Delphi Company must sell during the next fiscal year in order to break
even is

A. 22,500.
B. 20,930.
C. 18,140.
D. 25,500.

A. The formula to find the answer is the following: Total fixed cost / contribution margin per unit = $450,000 /
$20 contribution margin per unit ($36 sales price - $16 variable costs) = 22,500 units

B. See correct answer.

C. See correct answer.

D. See correct answer.

Question 150 - CMA 692 4-28 - Decision Making

Power Systems, Inc. manufactures jet engines for the United States armed forces on a cost-plus basis. The cost of a
particular jet engine the company manufactures is shown as follows.

Direct materials $200,000


Direct labor 150,000
Overhead:
Supervisor's salary 20,000
Fringe benefits on direct labor 15,000
Depreciation 12,000
Rent 11,000
Total cost $408,000

If production of this engine were discontinued, the production capacity would be idle, and the supervisor would be laid
off. When asked to bid on the next contract for this engine, the minimum unit price that Power Systems should bid is

A. $385,000.
B. $397,000.
C. $408,000.
D. $365,000.

A. In this problem want to calculate all avoidable cost. Therefore, the minimal unit price that Power Systems
should bid is $385,000 ($200,000 DM + $150,000 DL + $20,000 Supervisors salary + $15,000 Fringe benefits on
DL).

B. This answer includes depreciation which is an unavoidable cost, and should not be included.

(c) HOCK international, page 91


Part 3 : 11/11/10 07:42:55

C. This answer includes unavoidable cost (depreciation and rent) that should not be included.

D. This answer does not include the Supervisor's salary who would be laid off if production of the engine were
discontinued. This is an avoidable cost and should be included in the calculation.

Question 151 - CMA 691 4-9 - Decision Making

A company's approach to a make-or-buy decision

A. Depends on whether the company is operating at or below normal volume.


B. Involves an analysis of avoidable costs.
C. Should use absorption (full) costing.
D. Depends on whether the company is operating at or below breakeven.

A. Operating at or below normal volume is not relevant to the decision.

B. In its analysis a company would be looking only at relevant cost, not total cost. Relevant cost in a
make-or-buy decision would include only avoidable cost (variable and fixed).

C. Absorption costing includes all variable and fixed manufacturing cost, of which, not all are relevant to the decision
whether to make-or-buy decision.

D. Breakeven analysis is not relevant to the decision.

Question 152 - CIA 596 II-2 - Decision Making

An organization's executive committee, meeting to solve an important problem, spent 30 minutes analyzing data and
debating the cause of the problem. Finally, they agreed and could move on to the next step. Possible steps in the
creative problem-solving process are listed below. Which step should the committee perform next?

A. Generate alternative solutions.


B. Consider the reaction of competitors tovarious courses of action.
C. Identify the problem.
D. Select a solution.

A. The steps in the problem-solving process are: (1) Identify the problem (2) Generate alternative solutions (3)
Select a solution (4) Implement and Evaluate the selected solution.

B. This answer would be part of Step 4 (Implementing and Evaluating the selected solution).

C. This is incorrect because identifying the problem is the first step in the problem-solving process.

D. Selecting the solution is the third step in the problem-solving process.

(c) HOCK international, page 92

Vous aimerez peut-être aussi